Download as docx, pdf, or txt
Download as docx, pdf, or txt
You are on page 1of 57

DISEASES oral mucosa

1. Patient D. , aged 50, complains of pain while receiving spicy food for two weeks Examination :
buccal mucosa hyperemia , erosion and are silvery white papules in a grid. What are the most likely
diagnosis .
A. Leukoplakia , erosive and ulcerative form
B. Leukoplakia , towering form
C. Lichen planus, erosive and ulcerative form
D. Lichen planus, exudative hyperemic form
E. Lupus erythematosus, erosive form
 
2 . Patient N. , 40 years old , complains of periodic appearance of " sores " in the mouth, sick for 4
years, with exacerbations 3-4 times a year . Suffers chronic colitis . On examination : on the right
buccal mucosa erosion has rounded shape measuring 0.5 cm , covered with a grayish bloom
giperemirovanym circuit. What is the likely diagnosis?
A. traumatic ulcer
B. Chronic recurrent herpes
C. aphtha Setton
D. Chronic recurrent aphthous stomatitis
E. Secondary recurrent syphilis
  
3 . Patient '21 is explicit manifestation of necrotic stomatitis in the frontal area of the jaws. For 3
months noted weakness , fever to 37.50 C, drastic weight loss , diarrhea. On examination, her face
pale , lymph nodes are enlarged , painless on palpation. Place a preliminary diagnosis.
A. AIDS
B. Acute leukemia.
C. Vincent stomatitis .
D. Hypovitaminosis C.
E. Agranulocytosis .
  
4 . Patient 25 years old, complained of the presence of ulcers on the tongue. OBJECTIVE: on the
side surface of the tongue ulcer oval, 1 cm in diameter with raised edges and tight - elastic infiltrate
at the base , palpation painless . Lymph nodes on the affected side increased , dense , painless. What
is the most likely diagnosis ?

A. cancerous ulcer
B. Miliary tuberculosis ulcerative
C.Afta Setton
D. Shankriformnaya pyoderma .
E. primary syphilis

5 . Patient 45 years old accountant of private enterprise , complains of dry lips , sometimes burning,
the appearance of scales , which she skusyvaet . OBJECTIVE: dry lips on the red border has large
scales resembling mica platelets , their edges raised. After removing the scales of erosion not only
marked hyperemia . What is the most likely diagnosis ?

A. Weather cheilitis.
B. Exfoliative cheilitis.
C. Eczematous cheilitis.
D. Allergic contact cheilitis.
E. Actinic cheilitis.

6. Patient K., 23 years old, asked to reorganize the mouth. The examination revealed halitosis
patient. Oral mucosa , especially of the lower lip , floor of the mouth , pharynx and tonsils swollen
and clearly hyperemic, covered with purulent greenish patina with an unpleasant odor . Gums
bleeding, swollen, cyanotic hue ; gums of pus on palpation . Microscopy revealed : a significant
number of leukocytes , bean-shaped cocci arranged in pairs .
A. gonorrheal stomatitis
B. Acute catarrhal gingivostomatit
C. Generalized periodontitis, initial degree , acute course
D. Acute necrotizing ulcerative gingivostomatitis
E. diphtheria

7. Patient C, 34goda . Complaints about the presence of an ulcer on his left cheek , the appearance
of which connects with the occasional biting during meals 2 weeks ago. Objectively: the mucous
membrane of the left cheek has a painful ulcer with jagged edges , covered with a grayish bloom
surrounding mucosa hyperemic , with small hemorrhages around the ulcer. Submandibular lymph
nodes are enlarged and painful left . Diagnose .

A. Acute herpetic stomatitis, mild form


B. Chronic recurrent aphthous stomatitis
C. Milliarno ulcerative tuberculosis RBCU
D. Traumatic ulcer of the left cheek
E. Initial syphilophyma left cheek

8. Patient A. , 57y.o. . Complaints of blanching and burning sensation at the site of lesions of the
red border of the lower lip , which is a long time injured edged destroyed 11 tooth. OBJECTIVE: on
the red border of the lower lip in contact with the tooth 11 is allocated portion hyperkeratosis
polygonal shape , size 4 x 9 mm with clear contours , the lesion is flat, does not protrude above the
surface of the surrounding tissue and covered with thin scales grayish -white color. Regional lymph
nodes are not palpable . Diagnose .

A. Narrow premalignant hyperkeratosis


B. Leukoplakia , a planar shape
C. Precancer warty red border
D. Cancer of the red border of the lower lip
E. Lupus vermilion border

9. Patient K., 65 years , complains of burning and dryness of mucous membranes in the last 2.5
months at lamellar prosthesis , which is used by 4 years. Objectively: the mucosa of the hard palate
in the prosthetic bed hyperemic, swollen, dry, with a minor amount of white bloom . On the back of
the tongue papillae atrophy . Blood test - no change . In what direction diagnosis requires further
examination ?
 
A. Allergic contact stomatitis
B. Glossodiniya
C. leukoplakia
D. Chronic atrophic candidiasis
E. Lichen planus

10 . Patient L., 17 years old, during a rehabilitation doctor revealed on his cheeks on the line
between the teeth and the mucous membrane of the lower lip with a shiny white spots ( velvet )
surface. The lesions resemble matserirovanuyu spongy surface that does not hurt and is not
scraped . Lymph nodes are not palpable . Cytological examination : leukocytes , normal microflora ,
parakeratosis . Your diagnosis .
A. soft leukoplakia
B. Secondary syphilis
C. Chronic hypertrophic candidiasis
D. Systemic lupus erythematosus
E. Narrow peredrakovy hyperkeratosis

11. Male 31 years old, complains of dryness, burning back of the tongue caused about a week ago ,
worse reception irritating foods . Two weeks was in the hospital for pneumonia , was taking
antibiotics . Who does not take drugs . OBJECTIVE: oral mucosa hyperemic dry. On the back of
the tongue and palate gray-white curd easily removable plaque. Saliva stretches threads per spatula.
What is likely the preliminary diagnosis ?
A. Chronic hyperplastic candidiasis
B. Acute atrophic candidiasis
C. Acute pseudomembranous candidiasis
D. medical stomatitis
E. Chronic atrophic candidiasis

12. Girl , 18, complained of a sharp pain in the mouth , fever up to 38.30 C. After the examination
was the preliminary diagnosis : acute herpetic stomatitis . That will be detected by cytological
examination of material from the surface erosion .
A. acantholytic cells
B. atypical cells
C. Giant multinucleated cells
D. Cells Pirogov - Lanhgansa
E. The elements of the reticuloendothelial system

13. Patient , 19 years old , was diagnosed with acute necrotizing ulcerative stomatitis Vincent .
Choose the most effective set of drugs for general treatment.
A. Aspirin + + metronidazole + suprastin kordiamin
B. Aspirin + metronidazole + ascorbic acid
C. Erythromycin + + suprastin calcium gluconate
D. Erythromycin metronidazole + + + suprastin ascorbic acid
E. Erythromycin + ascorbic acid + + phenylbutazone kordiamin

 14. The patient , aged 30, complained of the presence of painful education in the mouth.
Objectively: the mucosal surface of the lower lip is defined epithelial defect with clear oval outlines
the size of 0.3 x 0.5 mm, surrounded by a rim of hyperemia and covered with fibrinous coating.
Which element of the lesions in this patient?
A. excoriation
B. erosion
C. aphtha
D. ulcer
E. crack

  15. The patient , aged 65, complained of a feeling of tightness, roughness on the mucous
membrane of the cheeks in the corners of the mouth. When viewed in the corners of the mouth
mucosa changes are detected as a grayish - white film in the shape of a triangle . Education at
poskablivanii not removed . Diagnosed flat leukoplakia . Color of the lesion caused by the
pathological process follows :
A. papillomatosis
B. acanthosis
C. hyperkeratosis
D. dyskeratosis
E. fibrosis

  16. The patient , aged 50, complains of the presence of surface roughness , a change of the buccal
mucosa . When viewed on the mucous membrane of the cheeks marked rash slightly elevated above
the level of the mucosa of gray-white color, do not shoot at poskablivanii . Merge to form a shape in
the form of lace . General state of the patient is not changed. Place a preliminary diagnosis.
A. Typical form of lichen planus
B. Acute pseudomembranous candidiasis
C. Leukoplakia , verrucous form
D. papular syphilis
E. Alopecia lupus erythematosus

  17. After clinical and laboratory examination of the patient , 60 years old , diagnosed with "
acantholytic pemphigus ." What are the primary morphological element lesions in this disease.
A. intraepithelial bubble
B. vesicle
C. pustule
D. subepithelial bubble
E. blister
            
18. The patient, aged 42, complains of bleeding gums and a metallic taste in the mouth. Of history -
the patient works as a storekeeper in the printing . Objectively: skin patient grayish earthy hue. In
the oral cavity - gingival margin swollen , amid congestion around the necks of front teeth bluish-
black strip . On the mucosa of the lower lip marked single spot of similar color . Preliminary
diagnosis ?
A. intoxication stomatitis
B. mercurial stomatitis
C. bismuth stomatitis
D. lead stomatitis
E. catarrhal stomatitis
19. The patient , aged 17, complains of a headache , fever , weakness , pain, oral mucosa , bleeding
gums . Ill three days. OBJECTIVE: pale face , red border of lips dry , t body - 380s , halitosis .
Lymph nodes are enlarged , painful. On the buccal mucosa in the lower teeth ulcers eighths in
diameter of 1-2 cm , with the transition to the gingival papillae , covered with gray patina , bleeding
gums . Determine the etiology of this disease ?
A. mushrooms
B. treponema pallidum
C. Streptococci staphylococcal microflora
D. Koch's bacillus
E. Fuso - spirilyarny symbiosis

20. The patient , aged 20, complains of general weakness, fever, headache. Sick week. Objectively:
the lymph nodes are enlarged, painful on palpation , t - 37,80 C, oral mucosa hyperemic, edematous
. On the mucosa of the lips , palate , gums , cheeks polycyclic group erosion incorrect form, coated
with a grayish- white coating , painful. What drugs appoint primarily in the topical treatment in the
early days of the disease.
A. vitamins
B. corticosteroid ointment
C. antiviral
D. antiseptics
E. dyes

21. Patient S. , 23 years old, complains of a cosmetic defect in connection with the appearance of
sores on the red border of the lower lip , which did not respond within two weeks . Objective : to
unmodified red border of the lower lip - ulcer rounded shape with a diameter of 2 mm with raised
equal edges; its bottom - red meat , dense , shiny with " greasy touch " ; palpation painless , with
chondroid infiltrate the base. Regional lymph nodes are enlarged, chondroid density , painless,
mobile . What is the most likely diagnosis ?
A. primary syphilis
B. Dekubitalnaya ulcer
C. cancerous ulcer
D. allergic cheilitis
E. tuberculous ulcer

22. Patient K. , 20 years old , asked the dentist to treat damaged teeth . During the inspection,
revealed on palatal handles , soft palate , tonsils, tongue rounded painless papules gipermirovannym
infiltrated with a whisk , measuring 7 mm in diameter. Regional lymph nodes are enlarged,
chondroid density , painless , mobile. What should be the tactics of the doctor ?
A. Send to a consultation venereologist
B. Conduct bacteriological examination
C. Send to a consultation with an ENT - doctor
D. To reorganize the mouth
E. Send for laboratory testing
23. The patient , aged 16, complained of bleeding gums . From history : frequent nosebleeds ,
general weakness . OBJECTIVE: pale skin and mucous membranes of the mouth , on the buccal
mucosa , tongue and soft palate - multiple petechiae .
Analysis of blood : red blood cells - 3.1 million , leukocytes - 2,9  109 / L , platelets - 80  109 /
L, blood clotting time - 9 min , color index - 0.9 , ESR - 48 mm / h , hemoglobin 94 g / L
A. disease Vakeza
B. agranulocytosis
C. iron deficiency anemia
D. chronic lymphocytic leukemia
E. disease Verlgofa

24. The patient , aged 45, complains of a burning sensation , tightness buccal mucosa , roughness,
unpleasant sensation . Objectively: the red border on the lower lip and buccal mucosa hyperemia on
the line between the teeth on the right there is a sealed portion grayish-white , above the level of the
mucous membrane in the form of white warty growths . Dental plaque . Diagnose .
A. Lichen planus
B. verrucous leukoplakia
C. Leukoplakia , a planar shape
D. Chronic hyperplastic candidiasis
E. galvanic stomatitis

25. The disease develops without premonitory phenomena manifested rash 1-2 aphthas size 5-8 mm
, rounded, surrounded by hyperemia and whisk furry yellow- gray. The recurrence rate with a
certain regularity , duration - 7-10 days . What disease is characterized by ?
A. lupus erythematosus
B. herpetic stomatitis
C. candidiasis
D. papular syphilis
E. Chronic recurrent aphthous stomatitis

26. The patient, 23 years old, complains of bleeding gums , fatigue , malaise . Symptoms appeared
recently . Objectively: skin and oral mucosa pale . Gums bleed when touched . On the mucous
membrane of the cheeks , lips and palate petechiae . What laboratory studies should be performed
for diagnosis ?
A. Blood sugar
B. Complete blood count
C. Definition of vit. C. Blood
D. Identify bleeding time
E. immunogram
27. The patient , 25 years old , complains of an unusual kind of language . Objectively: the back of
the tongue on the observed longitudinal and transverse grooves, which are stored in the depths of
filiform papillae . What is the likely diagnosis ?
A. Pleated language .
B. Desquamative glossitis .
C. Hairy tongue .
D. Rhomboid glossitis .
E. forked Tongue
28. Patient Ts , 27 years old complains of the appearance of rash in the mouth, on the lips and skin ,
general weakness , fever . Sick for 3 years , recurrent notes in spring and autumn . Objective : to
hyperemic and edematous mucosa of the mouth - bubbles painful erosions covered with gray -
yellow fibrinous coating. On the lips - hemorrhagic crusts . On the face, neck and dorsum of the
hand maculo - papular rash. Select the most likely diagnosis ?
A. herpetic stomatitis
B. pemphigus
C. Exudative erythema multiforme
D. Bulezny pemphigoid
E. Duhring disease

29. Patient B. , 34 years old , sick 5 days ago , when suddenly a short time there was a formless
swelling of the lips. OBJECTIVE: lips are enlarged , irregularly compacted painless. Labial gland
unchanged. Pathohistological picture : productive inflammation with proliferation of connective
tissue and the presence of granulomas in it with clear boundaries, consisting of lymphoid cells and
epiteliodnyh . Select the most likely diagnosis.
A. Glandular cheilitis purulent form
B. Glandular cheilitis edematous form
C. Granulomatous makroheylit Miescher
D. Limfedematozny cheilitis
E. Melkersson -Rosenthal syndrome

30 . Woman, 21 years old, referred to the clinic with complaints of dryness, itching and burning
lips. The appearance of these sensations is associated with the use of a new lipstick. Objectively :
the red border of lips dry, flushed , edematous , determined slight peeling . Overall condition is not
violated. Which of these ointments you assign this patient ?
A. heparin
B. Dibunolovuyu
C. prednizolonovuyu
D. Tebrofenovuyu
E. eritromitsinovaya

31. Man, 24 years old, complains of pain , bad breath , fever up to 380s . Male pale adynamic .
Regional lymph nodes are enlarged and painful . Gums swollen, hyperemic, ulcerated , covered
with necrotic plaque. On the teeth - excessive tartar . What additional research is needed to carry
out this sick in the first place?
A. Blood sugar
B. Microscopy plaque from the gums
C. HIV testing - infection
D. Overall full blood count
E. X-rays of the jaws

32. Man, 55 years old, complains about the presence of whitish layers on the left buccal mucosa ,
which appeared three months ago. Objectively: the mucous membrane of the left cheek on the line
between the teeth, close to the corner of the mouth portion is determined by dense growths of hilly
white size 2.0-1.5 cm with clear margins , rising above the level of the mucosa , painless on
palpation. Surrounding mucosa lesion is not changed. Crowns 34 , 35 and 36 destroyed. Choose the
most efficient method of local treatment of this patient.
A. Oil solution of vitamin A
B. ectylotic
C. surgical excision
D. corticosteroids
E. antifungals

33. Patient S. , 48 years old , complains of feeling of tightness of the oral mucosa , pain during
mealtimes . When viewed from buccal mucosa hyperemic single erosion to 3 mm in diameter,
surrounded by silvery-white papules in a fern leaf . Diagnose :
A. Lichen planus, erosive and ulcerative form
B. Leukoplakia , erosive and ulcerative form
C. Lichen planus, exudative hyperemic form
D. Lupus erythematosus, erosive form
E. Leukoplakia warty form

34. The patient , aged 48, complains of burning , dryness of the oral mucosa , viscous saliva , pain
when eating . For 18 years, enjoys dentures . On examination, buccal mucosa , tongue, palate and
swelling, hyperemic , covered with a minor amount of plaque that is poorly removed . Diagnose :
A. Chronic atrophic candidiasis
B. Lichen planus, exudative hyperemic form
C. Lichen planus, erosive and ulcerative form
D. Leukoplakia , erosive form
E. Acute pseudomembranous candidiasis

35. Patient V. , power line installer complains of a feeling of tightness, formation of scales on the
lips in the autumn-winter period . Examination revealed a red border of lips unevenly hyperemic
infiltrated , covered with small scales. Diagnose :
A. Weather cheilitis
B. Exfoliative cheilitis, dry form
C. Actinic cheilitis, exudative
D. Actinic cheilitis, dry form
E. Allergic contact cheilitis

36. During medical checkups student A. , 20 years old , diagnosed chronic candidiasis of the
mouth , generalized lymphadenopathy . In the history throughout the year herpes simplex . Body
temperature constantly rises to 37.40 C - 37.50 C weight in the last month decreased by 8 kg. What
disease can testify to this symptom ?
A. kissing disease
B. acute leukemia
C. Acquired immunodeficiency syndrome
D. Secondary syphilis
E. tuberculosis
37. Woman, 60 years old, complains of burning lower lip , increasing during the meal and
conversation. OBJECTIVE: on the face (nose and cheeks), erythema in the form of a butterfly. On
the red border of the lower lip bright red lesion that covered whitish- gray scales in the center of
lesion atrophy, on the periphery - areas turbidity epithelium. Diagnose :
A. leukoplakia
B. lupus erythematosus
C. Lichen planus
D. exfoliative cheilitis
E. warty precancer

38. The patient , 60 years old , complains of pain during eating , the presence of erosion in the oral
mucosa . First ill more than a month ago. Objective : to unmodified gum shell , soft palate large
erosion of bright red color . Mucous and peels easily injured when rubbing . In smears found
Ttsanka cells . Determine diagnosis.
A. pemphigus vulgaris
B. pemphigus vegetans
C. Benign Pemphigus neakantoliticheskaya
D. Exudative erythema multiforme
E. Bullous lichen planus

39. The patient , aged 50, complains of foreign body sensation on the tongue, with the severity of
the conversation, dryness in the mouth . OBJECTIVE: on back of the tongue - elongated to 5 mm
dark filamentous papillae . What is the most likely diagnosis ?
A. rhomboid glossitis
B. desquamative glossitis
C. pleated language
D. glossalgia
E. Black " hairy " tongue

40. The patient , 62 years old, complains of recurrent blisters in the mouth. When viewed from the
side surface of the left side first bubble size 8 x 8 mm, with hemorrhagic content . Palpation
painless. Nikolsky negative symptom . Blood pressure 200/120 mmHg . What is the most likely
diagnosis ?
A. pemphigus vulgaris
B. angioma language
C. Vesico - vascular syndrome
D. Exudative erythema multiforme
E. Dermatitis herpetiformis Duhring

41. The patient , 60 years old , complains of burning , discomfort in the language. On examination
of the tongue looks like " varnish " , filiform papillae are atrophied , raided the back of the tongue is
missing . What is the symptom of the disease is the first defeat in this patient ?
A. hyperacid gastritis
B. hypoacid gastritis
C. Peptic ulcer
D. enterocolitis
E. stomach cancer
42. Patient A. , aged 55, complains of itching, burning, rashes on the skin of the neck , ears , as well
as pain and ulceration in the mouth on the cheeks , palate . Ill two months . ago, when there was a
strong burning, itching and small blisters on the skin , a week felt the pain and burning sensation in
the mouth. From history we found that 2 years ago had an allergic reaction to potassium iodide .
Remove the drug Demerol and appointment - all phenomena are rapidly decreased. OBJECTIVE:
when viewed from the soft palate , cheeks on both sides - merged erosion bright red with scalloped
edges , slightly painful. SB around erosions swelling, hyperemic . Nikolsky negative symptom .
Jadassohn sample positive. On the skin of the neck , ears , polymorphic elements ( erythema ,
vesicles , crusts ) . Diagnose .
A. Exudative erythema multiforme
B. pemphigus vulgaris
C. Dermatitis herpetiformis Duhring
D. allergic stomatitis
E. Lichen planus ( pemfigoidnaya form)

43. The patient , aged 37, complains of an unusual type , burning, sore tongue when eating ,
weakness, malaise, dizziness. Changes noted during the month. OBJECTIVE: atrophy of the
filiform papillae of the language in back of the tongue - Striping hyperemia with different shades .
In blood test : Er. -2 , 7 • 10 12 / l, Hb - 90 g / l, ts.p.1 , 2 , A -4 , 6 • 10 9 / L , n - 3 \ % a - 57 \ % l -
34 \ % m - 4 \ % e - 9 \ %, ESR - 12 mm / hour. What will be the defining indicator of blood at
diagnosis ?
A. color index
B. leukocytes
C. erythrocytes
D. hemoglobin
E. ESR

44. Patient A. , 43, complained of cracks in the corners of the mouth , which are covered with crusts
, dryness, redness and peeling lips, photophobia, and lacrimation, weakness, loss of appetite ,
decreased performance , peeling skin , burning sensation in the tongue. What causes this disease ?
A. Hypovitaminosis B1
B. Hypovitaminosis RR
C. Hypovitaminosis B2
D. Hypovitaminosis B6
E. hypovitaminosis A

45 . Patient 24 years, complains of severe pain in the mouth , general weakness , headache , joint
pain , fever to 38.9 C. ill 2 days ago after hypothermia . OBJECTIVE: swollen lips on the red
border - hemorrhagic crusts . On the oral mucosa on the background diffuse hyperemia - large
confluent erosions and ulcers covered with yellowish - gray patina , sharply painful , bleed when
touched . Conjunctivitis . On the skin of the hands and forearms - erythematous patches up to 1.5
cm in diameter with bubbles in the center. What naibol6ee likely diagnosis ?

A. medical stomatitis
B. Exudative erythema multiforme
C. Behcet's syndrome
D. Lyell's syndrome
E. Stevens - Johnson

46. Patient 25 years old foci of necrosis along the gingival margin on both jaws . Facial skin sallow
hue. Looks older than his years . Submandibular, chin , neck and axillary lymph nodes are
enlarged , painless, mobile . Leads a hectic life, using drugs . In the last month notes weakness,
constant increase in body temperature to 37,3-37,50 C, diarrhea, weight loss of 15 kg. What is most
likely the preliminary diagnosis ?
A. AIDS
B. acute leukemia
C. hypovitaminosis C
D. gingivitis Vincent
E. agranulocytosis

47. By dentist turned 35 years old patient with complaints of a purple color formations on the oral
mucosa . After the examination, diagnosed with Kaposi's sarcoma. AIDS. Kaposi 's sarcoma, as one
of the characteristic clinical signs of AIDS is formed from:
A. blood vessels
B. lymphangion
C. nervous tissue
D. muscle tissue
E. Bone tissue

48. Patient 23 years , visit the second day of the disease , diagnosed as acute herpetic stomatitis
moderate severity. What ointment for the treatment of lesions elements you assign in the first place?
A. antiviral
B. Antibiotikosoderzhaschuyu
C. hormonal
D. sulfa
E. Fermentosoderzhaschuyu

49. The patient, 27 years old, complains of painful education in the language within a month ,
cough with sputum , fever up to 37.20 - 37.40 S. eve suffered SARS. Course conducted penicillin is
not effective. Objectively: the back of the tongue on a shallow ulcer , the bottom of which is
satisfied with small abscesses granulations on the periphery . Regional limfouzdy enlarged, painful,
soldered into packets. What preliminary diagnosis established dentist ?
 
A. Miliary - ulcerative tuberculosis
B. lupus
C. syphilis
D. candidiasis
E. leprosy

50 . During the baseline medical examination at the student 17 years , the dentist revealed by the
line between the teeth and the surrounding areas whitish , macerated mucosa. Epithelium unevenly
sluschen has fringed view lifted at poskablivanii . What is the most likely diagnosis ?
A. Lichen planus
B. chronic candidiasis
C. soft leukoplakia
D. lupus erythematosus
      E. Erosive leukoplakia

51. Patient 38 years old, complained of dryness and peeling lips. During the month smeared lips
indifferent ointment , there was no improvement . Objectively: the red border of the lower lip bright
- red , moderately infiltrated , covered tightly seated whitish - gray scales when trying to remove
that there is soreness and bleeding . On the periphery of the hearth - clouding epithelium as white
strips and depressions in the central portion . What is the most likely diagnosis ?
A. candida cheilitis
B. Lichen planus
C. leukoplakia
D. exfoliative cheilitis
E. Systemic lupus erythematosus

52. Patient 50 years cheeks and palate mucosa erosion with scalloped contours located on slightly
edematous and hyperemic background that appeared after treatment with oral mucosa 2% iodine
solution . Symptom negative Nikolsky , acantholytic cells do not. Any erosion at the site of multiple
bubbles with transparent content . On the skin - itchy rash abundant polymorphic . In blood and
contents of bubbles - eosinophilia . Diagnose .
A. Dermatitis herpetiformis Duhring
B. Bullous pemphigoid Lever
C. Stevens-Johnson syndrome
D. Exudative erythema multiforme
E. allergic reaction

53. Patient 17 years , complains of malaise, pain in the mouth, bad breath . Sick for 2 days.
OBJECTIVE: body temperature 38.70 C. Regional lymph nodes are enlarged on the right , painful
mobile. Mucosa hyperemic, edematous papilla bleeding at the gingival margin - gray - white
patches in the area 48 - ulcer with jagged edges , painful when touched , covered with dirty - gray
patina . What microflora most likely be detected by microbiological examination ?
A. Fuzobakterii and spirochetes
B. Actinomycetes and diphtheria bacillus
C. Actinomycetes and fuzobakterii
D. Trichomonas and spirochetes
E. Fungi of the genus Candida and Trichomonas

54. Patient 57 years old, complains of the presence of whitish layers on the left buccal mucosa ,
which appeared about a year ago . Giperatsidnym suffering from gastritis . OBJECTIVE: on the
buccal mucosa and in the corner near the mouth is determined plot dense whitish lumpy growths
measuring 1.5 x 2 cm with clear contours , towering above the surrounding intact mucosa , painless
on palpation. Which of the following is the most appropriate for the local treatment of this disease ?
A. ectylotic
B. anti-inflammatory drugs
C. surgical excision
D. antifungals
E. corticosteroid agents

 55 . The patient K., 70 years on the alveolar ridge mucosa doctor discovered multiple growths
associated with wearing the denture . After examination diagnosed papillomatosis . Give
histopathologic definition of the state.

A. Thickening thorny layer cells , epithelial outgrowths elongation mezhsosochkovyh


B. Thickening of the stratum corneum
C. Sprawl papillary layer of the lamina propria towards epithelium
D. Proliferation of the granular layer of the epithelium
E. Absence of the granular layer , the preservation of the nuclei in the cells of the stratum corneum.

56. Patient 57 years old, complained of pain , difficulty opening the mouth. Was diagnosed with
leukoplakia ulcerative form . 12 days after the course of treatment noted the appearance of flabby
granulation processes and strengthening keratinization at the bottom ulcers. What the study of the
following must take place in the first place?

A. cytological
B. bacterioscopic
C. bacteriological
D. fluorescent
E. Stomatoskopiya

57. Patient M. , aged 55, complains of swelling and bleeding gums, especially when taking solid
food , fatigue , itchy skin, sometimes numbness mucosa. Have a history of bleeding from the nose .
The facial skin is dark cherry hue. Objectively : the red border of lips , mucous membrane of the
lips, tongue, cheeks bright red . Hyperemic interdental papillae have a dark cherry color bleed when
touched . Marked cyanosis soft and pale staining of the hard palate (symptom Cooperman ) . What
is the most likely diagnosis ?
A. disease Vakeza
B. hyperchromic anemia
C. agranulocytosis
D. disease Verlgofa
E. acute leukemia

58. Patient 55 years old, complains of pain during a meal , the presence of erosions on the oral
mucosa . First ill more than a month ago. Objective : to unmodified mucosa of the soft palate and
shackle large erosion of bright red color . Mucous and peels easily injured when rubbing . Minor
pain . In smears Ttsanka cells . Diagnose :
A. pemphigus vulgaris
B. Neakantoliticheskaya pemphigus
C. Benign Pemphigus neakantoliticheskaya
D. Exudative erythema multiforme
E. Bullous lichen planus
59. Patient 49 years in the treatment of lichen planus ( erosive and ulcerative form ) for 1 month
failed to reach epithelialization sections defeat. What is the best tactics of the doctor in this case?
A. Surgical treatment of an urgent histological examination
B. Purpose UHF , laser
C. Repeated treatment with the use of other means
D. Appointment imunostimuliruyuschey therapy : vitamins
E. Obkalyvanie lesion honsuridom

60 . Patient 38 years old, complains of pain in the mouth when eating and from different stimuli ,
poor overall health. OBJECTIVE: painful erosions on the oral mucosa , which is on the edge of tire
scraps bubbles. Surrounding mucosa moderately inflamed. Notes on the back skin erosion, painful
to touch clothing. What additional testing is necessary for definitive diagnosis ?
A. virological
B. cytological study
C. immunological research
D. Complete blood count
E. bacterial study

61. Patient 45 years old, complains of feeling of tightness buccal mucosa , roughness, discomfort.
OBJECTIVE: on the buccal mucosa whitish gray areas slightly above the level of the mucous prone
to draining a lacy pattern , localized in the posterior parts of the oral cavity. When poskablivanii not
removed . What disease is characterized by such clinical picture ?
A. Lichen planus
B. Secondary syphilis
C. verrucous leukoplakia
D. Chronic hyperplastic candidiasis
E. lupus erythematosus

62. The patient was 18 years old, diagnosed as a folded tongue. What symptom syndrome can have
this malformation ?
A. syndrome Stivensona_Dzhonsona
B. Behcet's syndrome
C. Syndrome Stanton - Kapdepona
D. Melkersson -Rosenthal syndrome
E. Sjogren's symptom

63. Patient 18 years , complains of bleeding gums , changing their appearance. Suffered severe
SARS . When viewed papilla swollen , bluish- red cover on the crowns of the teeth ½, bleed much .
GPRS on multiple petechial krovoizmeneniya , in some areas at the gingival margin - ulceration . In
the analysis of blood there is no change . What vitamins to assign complex treatment of the patient ?
A. Retinol acetate + riboflavin
B. Ascorbic acid + routines
C. + Pyridoxine , thiamine
D. Rutin + nicotinic acid
E. Cyanocobalamin + thiamine
64. The patient , 38 years old , the driver complains of severe pain in the mouth, throat,
complicating meal , malaise , fever to 38.6 0C. After hypothermia sick during the day, night shift
should step up . Objectively: the gums in the lower front teeth , pharyngeal mucosa hyperemic ,
edematous, necrosis of the gingival margin and enlarged tonsils . Bacterioscopy : FUSO -
spirillyarny symbiosis. Blood test: Air - 4,5 • 10 12 / l, l - 7,2 • 109 / l, ESR - 18 mm / h , the
formula without features. What is the future tactics of the doctor - dentist?
 
A. Conduct treatment and issue a medical certificate
B. Provide treatment and to replace an
C. Send hospitalization
D. Conduct treatment and issue a certificate
E. Send to a consultation with an ENT - specialist

65 . Patient 50 years old, complains of burning , tingling, numbness of tongue , dry mouth .
Paresthesia occur spontaneously and tested during the meal. Ill the third year after the change of 2
bridges gold prostheses prostheses coated steel titanium nitrite . A year ago, the coated prostheses
removed, but the patient's condition has not improved . Antiseptic rinse, lubrication language
vitamins A, E and acupuncture brought no relief . OBJECTIVE: mucosa without visible pathology .
What is the most likely diagnosis ?
A. Glossodiniya
B. glossitis
C. candidiasis
D. medical stomatitis
E. B6 vitamin deficiency

66. Female 35 years , complains of periodic appearance of " sores " in the mouth for 3 years .
Relapses are repeated 4-5 times a year . " Sores " to allow for 7-8 days. OBJECTIVE: on the left
side of the tongue lesions detected element rounded shape 0.5 cm in diameter with clear smooth
edges , covered with a grayish bloom surrounded whisk hyperemia sharply painful when touched .
What is the most probable mechanism of the disease in a patient ?
A. autoimmune mechanism
B. allergic reaction
C. Viral infection
D. staphylococcosis
E. mechanical injury

67. The patient , 26 years old , complains of fever up to 380s , painful swallowing , bad breath ,
bleeding gums . Ill with rheumatism , take phenylbutazone , aminopyrine . OBJECTIVE: oral
mucosa pale, determined hemorrhages , areas of necrosis on the tonsils and the gingival edge in
posterior teeth left. Lymph nodes are enlarged , painless. What changes in the blood in this patient
most likely to occur ?
A. Increase in color index
B. myeloblastosis
C. neutropenia
D. thrombocytopenia
E. agranulocytosis
68. Patient 55 years old, complains of burning and discomfort in the language. About 5 years sick
hypoacid gastritis. What are the most likely to change the language for this patient?
A. hypertrophy of the papillae
B. Atrophy and flattening of tongue papillae
C. Erosion on the first side surfaces
D. furred tongue
E. folding language

69. Patient K., 63 years old, complains about the presence of erosion on the red border of the lower
lip , which has long existed, sometimes spontaneously epithelialized , and then appears in the same
place . The patient suffers from chronic gastritis giperatsidnym smokes . Objectively: the red border
on the right side of the lower lip is determined erosion of irregular shape with a smooth , polished
surface like a rich red color . Palpation painless , tissue seal at the bottom is missing . Lymph nodes
are enlarged . Select the most likely diagnosis :

A. Erosive form of leukoplakia


B. Abrasive prekantserozny cheilitis Mangonotti
C. Erosive- ulcerative form of lupus erythematosus
D. Erosive and ulcerative form planus
E. radiation cheilitis

70 . Patient K. , aged 36, complains of discomfort when eating , fever , malaise, swollen lymph
nodes . The patient smokes , uses alcohol moderately . Three years ago on a business trip in Africa.
OBJECTIVE: submandibular, cervical , axillary lymph node palpation increased, mobile , painless .
On the first lateral surface portions whitish formations resembling thread 3-4 mm in length . Which
of the following is necessary to clarify the diagnosis ?
A. Complete blood count
B. Analysis of blood RW
C. Revmoproby
D. A blood test for HIV
E. Analysis of blood sugar

71. G. 29 years old patient complains of a single slaboboleznennoe tumor-like formations on the
hard palate , which appeared about a month ago . OBJECTIVE: deep ulcer bluish - red color
surrounded by a massive dense infiltrate, covered with small granulations at the bottom reveals
bone sequestration . Diagnose :
A. Cancer of the hard palate
B.Abtsess palate
C.Tretichny syphilis
D.Travmaticheskaya ulcer complicated by osteomyelitis
E.Miliarno - ulcerative tuberculosis

72. Male 60 years , complains of pain in the lower jaw on the left side department. Pain increases
when eating . From history : 3 days ago the patient was put full denture on the lower jaw.
OBJECTIVE: mandibular complete denture . Left lateral region department transitional fold over
the edge of the prosthesis defect mucosa oval edges are covered with white bloom . When you
touch - a sharp pain . What is the diagnosis for this patient ?
A.Toksichesky stomatitis
B.Aktinomikoticheskaya ulcer
C.Sifiliticheskaya ulcer
D.Dekubitalnaya ulcer
E.Allergichesky stomatitis

73. Patient A. , 20 years old , was admitted to the hematology department with a diagnosis of "
acute leukemia " . The examination gingival margin in the lower front teeth necrotic , pale gums , in
the cervical area of the teeth blood clots . In blood test : HB -56 g / liter, E-3 , 2 • 10 12 / l,
tromb.100 • 10 9 / l, L-149 • 10 9 / l, ESR -42 mm / hour. What changes are most pronounced in the
mouth of this patient ?
A.Gemorragicheskie .
B. hyperplastic .
C. necrotizing .
D.Smeshannye .
E.Anemicheskie .

74. 58 years old patient complains of tumor presence on the lower lip , which appeared six months
ago. Over the past 20 days began to increase in size. Objectively: the red border on the lower lip
node semicircular gray- red , protruding above the surrounding tissue by 0.5 cm, delimited with a
funnel-shaped hollow in the center , made horny masses. On palpation dense, mobile , painless.
What is the most likely diagnosis .
A. Cheilitis Manganotti
B. Warty leukoplakia
C. Papilloma
D. Verrucose precancer
E. Keratoacanthoma

75 . Patient L. , 43, complains of the presence of rough spots in the corners of the mouth mucosa .
These manifestations appeared 7 months ago . Objectively: the teeth - a dense brown patina smoker.
Lots of white color can not be removed when poskablivanii . Mucosa bordering them unchanged.
Diagnose :
A.Leykoplakiya
B.Krasnaya erythematosus
C. Syphilis
D. Candidiasis
E. planus

76. Patient K. , age 33 , complains of pain and rash in the mouth , headache, aching muscles and
joints. 2 days before the rash appeared runny nose and sneezing . When viewed on the congested
mucous membrane of the lips , cheeks and hyoid region vesicles , erosions covered with fibrinous
coating. On the red border of lips dark - brown rind . Which of the following would be identified
cytology ?
A. Non-specific inflammation, fuzospirohetoz
B. Giant multinucleated cells
C. acantholytic cells
D. Bezyadernye epithelial cells
E. Acute nonspecific inflammation

77. The patient complains of a burning sensation of the oral mucosa , increased pain when receiving
spicy food. Recently treated for pneumonia . On examination of the oral cavity detected significant
white patches on the mucous membrane of tongue, cheeks . Plots uncovered bloom, bright
hyperemized brilliant. Call reliable diagnosis .
A. Candidiasis
B. Sjogren Sindh
C. planus
D. glossalgia
E. Allergic stomatitis

78. Patient A. , 19 years old , complains of sore lips, enlarged, crusting , impeding speech and
eating. OBJECTIVE: vermilion border hyperemic, edematous zone Klein abundant korochnye
bedding . Dentist What should elect to treat this patient ?
A.Miogimnastiku and massage
B.Smazyvanie lip cream containing biostimulators
C.Giposensibiliziruyuschuyu therapy
D. Irradiation vermilion border rays Bucca
E.Smazyvanie lips corticosteroid ointments

79. Patient B. , 40 years old contacted the clinic with complaints of general weakness , fever to 38.5
, difficulty eating due to severe pain in the oral mucosa . Given the existence of multiple ulcers in
different parts of the mouth, diagnosis : necrotizing stomatitis Vincent . Call microorganisms -
causative agents of disease?
A.Spiroheta Vincent
B.Simbioz actinomycetes and spirochetes
C.Simbioz saprophytes and spirochetes
D.Simbioz veretenopodobnoy rods and spirochetes
E.Trihomonady

80 . Patient G.51 year , appealed to the dentist with complaints of dryness , a metallic taste in the
mouth , worse when taking acidic foods. Prosthetic 2 months ago. Objectively : the red border of
lips dry , slightly cyanotic , single crown on 45 gold , 13 - Plastics bridges made of steel , in places
adhesions visible oxide films . On the chewing surface of 46 - from silver amalgam fillings . What
are the most likely preliminary diagnosis of the patient.
A.Kandidoz
B.Galvanizm
C.Toksichesky acrylic denture stomatitis on
D.Toksichesky stomatitis on metal prostheses
E.Allergichesky stomatitis

81. Female 58 years old when contacting the clinic complained of pain when eating , especially
hard . Considers herself to be sick for about three weeks . Sought help from the community : Rinse
your mouth with sage , dabbed karotolin - no improvement . Objective : to apparently intact mucosa
gums upper and lower jaws are determined extensive erosive surface is bright red ,
maloboleznennye palpation . When rubbing a cotton swab between mucosal erosion easily peels off
. What is the most likely diagnosis ?
A.Gerpetiformny dermatitis Dühring
B.Bullezny pemphigoid
C.Bolezn Bowen
D. " Climacteric " gingivitis
E.Vulgarnaya pemphigus

82. The patient, 55 years , complains of gum and nasal bleeding. Skin and mucosa hyperemic with
dark cherry shade . Cooperman positive symptom . What a picture of peripheral blood characteristic
of this disease :

A.E 3.5 x10 12 degree / l, 140 g Hb / l, 5 ESR mm / hour


B.E 5,5 x10 12 degree / L, Hb 150 g / l, ESR 12 mm / h
C. Br.e. 5.8 x10 12 degree / L, Hb 140g / l, ESR 12 mm / h
D.E 6.1 x10 12 degree / L, Hb 180g / L, erythrocyte sedimentation rate of 1 mm / h
E.E 4.5 x10 12 degree / l, 120 g Hb / l, 6 ESR mm / hour

83. Patient M. , 18 years old , diagnosed with acute herpetic stomatitis . Select for topical treatment
in the first 2-3 days of illness
A.Nesteroidnye anti-inflammatory drugs
B.Antibakterialnye drugs
C.Keratoplasticheskie drugs
D.Protivovirusnye drugs
E.Simptomaticheskoe treatment

84. Patient B., for 5 years every autumn , the disease begins with fever, pain in the joints. On the
skin of the upper and lower extremities cyanotic - pink spots with a diameter of 1-2 cm with a
bubble in the center. On the oral mucosa against the backdrop of extensive erythema and edema -
Bubbles , bubbles, sharply painful erosions , hemorrhagic crusts on the lips . Nikolsky negative
symptom . In blood test: A 12 • September 10 g / l, 4,7 • E December 10 g / L ESR 14 mm / h .
Select the most likely diagnosis :
A. Ulcer - necrotizing stomatitis Vincent
B. Chronic recurrent aphthous stomatitis
C. Neakantoliticheskaya pemphigus
D. exudative erythema multiforme
E. Chronic recurrent herpes

85 . 25 years old patient complains of availability ulcers on the tongue, which saw two weeks ago.
To the doctor did not address, was treated independently, but to no avail. The examination on the
back of the tongue on the background unchanged ulcer is 6 mm in diameter with splayed edges.
Smooth bottom , meat - red. Palpation painless, compacted base . Submandibular lymph nodes
dense , painless. Preliminary diagnosis :
A.Rakovaya ulcer
B.Tuberkuleznaya ulcer
C.Troficheskaya ulcer
D. Dekubitalnaya ulcer
E. Primary sifilioma language

86. Patient K. , 40 years old , with a preliminary diagnosis of acute pseudomembranous candidiasis
material taken for microscopic examination . Which answers confirm the diagnosis ?
A.Nepochkuyuschiesya yeast cells ;
B.Niti pseudomycelium ;
C.Pochkuyuschiesya yeast cells ;
D.Niti true mycelium ;
E.Drozhzhevye cells and accompanying flora .

87. Patient G. , 56 years old , complains of an increase of the lower lip and the pain of hot , sour,
salty , bitter , sticky lips in the morning. Lifelong periodically lip thickened , become lumpy . When
viewed from the lower lip increased. In the middle third zone Klein few small red dots with holes ,
of which exude transparent drops . Hyperemic red border , is covered with scales .
What is the most likely diagnosis ?
A.Ekfoliativny cheilitis dry form
B.Glandulyarny cheilitis, edematous form
C.Glandulyarny cheilitis , purulent form
D.Glandulyarny cheilitis fibrous form
E.Eksfoliativny cheilitis, exudative

88. Patient 37 years old, during a visit to the doctor - dentist filed a complaint on feeling the heat
and metallic taste in the mouth , increased salivation . On examination - giperemirovanna gums ,
swollen, bleeding when touched . At the gingival margin fringe gray - black, fetid layers identified
on the gums . Diagnose .
A.Svintsovy stomatitis
B.Yazvenno - necrotizing gingivitis
C.Rtutny stomatitis
D.Melanoz
E.Vismutovy stomatitis
89. Patient L. , 48 years old , complaining of loss of appetite , nausea , frequent diarrhea, mucosal
burning tongue and cheeks , dry , flaky skin , memory loss , apathy. In history : the patient has long
been a poor protein diet. Objectively: skin , especially on the cheeks , face, hands pigmented dry.
Language is increased in volume , swollen , bright - red , shiny, sharply painful. Select the most
likely diagnosis:
A.Gipovitaminoz RR
B. Hypovitaminosis C V.
C. Hypovitaminosis A
D.Gipovitaminoz B2
E E.Gipovitaminoz

90 . Patient 48 years old, complained of the appearance of sores on the mucous membrane of the
gums in the area of the molars , which gradually increases , the shakiness of the teeth in the affected
area . Cough. Objectively: the mucous membrane of the gums in the lower left molars have the 2nd
surface sharply painful sores that have saped edge . The bottom of the grainy appearance of ulcers
covered with yellowish , and in some places and pink granulation . Ulcers surrounded by hillocks .
Necks of the teeth exposed, noted abnormal mobility of the teeth. Regional lymph nodes are
enlarged, form packages. Preliminary diagnosis .
A.Dekubitalnaya ulcer
B.Ostry aphthous stomatitis
C.Sifilis
D.Infektsionny mononucleosis
E.Tuberkulez

91. Patient A. , 25 years old complains of general weakness, fever, bad breath , pain when eating .
In history : hypothermia. OBJECTIVE: pale skin , increased soreness lymph nodes , massive hard
and soft plaque . Ulcer covered with necrotic plaque with soft jagged edges on the buccal mucosa .
In haemogram - leukocytosis, left shift , a dramatic increase in ESR. Select the most likely
diagnosis:
A.Pervichny syphilis
B.Miliarno - ulcerative tuberculosis
C.Yazvenno - necrotizing stomatitis Vincent
D.Allergichesky stomatitis
E.Leykoz

92. 40 years old patient complains of availability of education on the lower lip . Objectively: the red
border on the lower lip education hemispherical shape, dense , measuring 4 x 5 mm above the level
of the lips for 3-4 mm , covered with thin , do not remove the scales. Regional lymph nodes were
not enlarged . What is the most likely diagnosis
A. Cutaneous horn
B. Papilloma
C. premalignant hyperkeratosis
D. Verrucose precancer
E. Keratoacanthoma

93. With a planned inspection at the dentist welder N. , 45 years old , which is on the "D" registered
for peptic ulcer disease , first discovered on the back portions diamond shape with a smooth
surface , red color, size 1,5-0,5 cm, located flush with the surrounding tissues , ahead of
circumvallate papillae , slightly tight when touched , painlessly. The doctor has diagnosed :
rhomboid glossitis, flat shape . What are the most likely cause of the disease.
A.Geneticheskaya liability to disease ( abnormal development of language )
B.Zabolevanie is a manifestation of gastric ulcer
C.V -based diseases are trophic disorders inervatsii dorsum of the tongue
D.Voznikaet as an occupational disease associated with the action of chromium and magnesium
E.Eto congenital disease caused by disordered embryogenesis language

94. Patient A. , 42 years old suddenly ill : a few hours swollen lower lip , acquiring the form of
proboscis that are separated from the teeth. Thickening lips uneven texture plotnoelasticheskoy .
Then came the front and type of migraine headache . On examination : the expansion of the optic
fissure , loss of muscle tone of the affected side of the face , drooping corner of the mouth , tongue
folded . Which most likely diagnosis ?
A.Sindrom Melkersson -Rosenthal
B.Parez facial nerve
C.Limfedematozny cheilitis
D.Otek Kwinke
E.Granulematozny Miescher cheilitis
    
95 . Patient B. , 45 years old , miner , complained of a burning sensation and pain in the mouth with
different stimuli . Objectively: the mucous membrane of the cheeks on the line between the teeth,
on the lateral surface of the tongue , land keratosis , towering above the mucosa , forming a wart
with lobed erosive surface. Which of the following is most likely diagnosis ?
A.Leykoplakiya , erosive form
B.Leykoplakiya , warty form
C.Leykoplakiya towering form
D.Krasny planus , erosive form
E.Krasnaya lupus erosive ulcerous form

96. Patient I. , 25 years old, single practically healthy . Complains of an unusual kind of language ,
a sense of discomfort . Objectively: the back of the tongue on the periphery areas found
desquamation round shape resembling dvuhkopeechnuyu coin. Palpation painless, submandibular
glands dense, enlarged, painless, not soldered to the surrounding tissues. For consultation to what
expert should send the patient?
A.Venerologu
B.Gematologu
C.Gastroenterologu
D.Dermatologu
E.Otolyaringologu

97. Patient 19 years, complains of itching of the lips , redness and peeling of the vermilion border .
On examination, marked infiltration , dry and cracked corners of the mouth and swelling of the
vermilion border . The skin of the face and neck dry, scaly , on the elbow bends and eskoriatsii
lihenizatsiya.Kakoy most likely diagnosis ?
A.Kontaktny allergic cheilitis
B.Ekfoliativny cheilitis, dry form
C.Atopichesky cheilitis
D.Ekfoliativny cheilitis exudative
E.Ekzematozny cheilitis

98. Patient B., 49 years old, the liquidators of the Chernobyl nuclear power plant , there is frequent
nosebleeds , bleeding gums positive symptom " harness ." Pre- diagnosed - Verlgofa disease .
Which of the indicators of a blood test will testify in favor of this diagnosis ?
A.Chislo leukocytes ;
B.Chislo reticulocytes ;
C.Gemoglobin ;
D.Chislo platelets ;
E.SOE .

99. Patient Z. , aged 30, complained of pain on swallowing , the presence of plaque in the mouth ,
discomfort when chewing in the right mandible . OBJECTIVE: GPRS hyperemic, edematous ,
copious greenish patina on the soft palate , the tonsils , the tongue in 46 chronic middle caries .
Bacteriological examination by Leffler found many diplococci . What tactics dentist in relation to
this patient?
A.Konsultatsiya otolyaringologa ;
B.Lechenie caries, oral hygiene ;
C.Konsultatsiya gastroenterologist ;
D.Konsultatsiya dermatology and venereology ;
E.Lechenie GPRS , antibiotics .

100 . Patient D. , 43 years old, suffering from diabetes, complains about the presence of plaque on
the tongue, pain, burning . OBJECTIVE: on the back of tongue and buccal mucosa tight gray patina
, it is difficult to shoot at poskablivanii . What method of collection of material be carried out to
confirm the diagnosis ?
A.Mazok - mark ;
B.Smyv ;
C.Soskob ;
D.Biopsiya ;
E.Punktsiya .

101. Patient B. , 32 years old, fell ill suddenly. 5 days ago in a short time there was a formless
swelling of the lips. Objectively: the edge of the lips are turned in the form of proboscis tissue
uneven lips are sealed . Pathohistological pattern consistent with the growth of non-specific
inflammation of the connective tissue and the presence of granulomas in it with clear boundaries,
consisting of lymphoid and epithelioid cells . What is the most likely diagnosis ?
A.Limfedematozny makroheylit ;
B.Otek angioedema ;
C.Granulematozny makroheylit Miescher ;
D.Sindrom Melkersson -Rosenthal ;
E.Glandulyarny cheilitis.

102. On examination of the oral cavity in a patient M. , 60 years old found that back of the tongue is
smooth, shiny , polished though . On the back and the tip of your tongue - painful , sharply limited
stripes and spots of bright red color . In the blood - a significant reduction in red blood cell count
and a high color index . What is the cause of this disease ?
A. Loss of iron in bleeding
B. Lack of iron in the body due to a deficiency in his food.
C. Lack of vitamin B12 ( cyanocobalamin ) .
D. And malabsorption disorder of iron metabolism
E. Endogenous bone marrow aplasia

103. Patient K., addressed to the dentist complaining of frequent bleeding from the mucous
membranes of the mouth and nose, indicates that the same problems were her father . OBJECTIVE:
There are multiple telangiectasia and angiomatoznye growths on the skin of the face and nasal
mucosa, cheeks, lips . Analysis of blood within normal limits.
A. disease Verlgofa
B. Rendu Osler disease .
C. disease Vakeza
D. Pituitary basophilia
E. Biermer's disease
104. Patient 72 years complained of the presence of neoplasms . OBJECTIVE: on the red border of
the lower lip is defined ulcer with raised thick edges , uneven , grainy bottom , covered with dense
gray patina . On palpation sealed at the bottom. Cytology observed polymorphism Anisocytosis ,
polinukleoz cells.
A. tuberculous ulcer
B. trophic ulcer
C. cancerous ulcer
D. traumatic ulcer
E. Keratoacanthoma .
105. Patient A. , 27 years old , sailor, complains of diarrhea over the past 5 weeks , an unusual type
of oral mucosa , heartburn, prolonged cough . Smokes. An objective examination of the oral cavity
established that a significant amount of soft white foci resembling flakes curds , with poskablivanii
easily removed , exposing the surface hyperemia . There polyadenylation . Set preliminary
diagnosis.
A. AIDS.
B. Acute pseudomembranous candidiasis .
C. Chronic hyperplastic candidiasis .
D. hairy leukoplakia
E. acute leukemia
106. A. The patient complains of unusual form GPRS . An objective examination revealed that the
gums, tongue , floor of the mouth , tonsils and pharynx covered with slimy greenish purulent
coating. Cytology plaque contains a large number of neutrophils . Set preliminary diagnosis.
A. Necrotizing gingivostomatitis Vincent .
B. Chronic hyperplastic candidiasis
C. Gonorrheal stomatitis .
D. Stomatitis .
E. Pemphigus, the leaf shape.
107. When routine inspection of the oral cavity of the patient revealed M. unilateral enlargement of
the tonsils . Objectively: the left amygdala copper- red , painless on palpation , dense. Around the
tonsils acute inflammatory hyperemia . On the opposite side of the amygdala intact . Patient no
complaints . No symptoms of intoxication . Set preliminary diagnosis.
A. tonsillitis
B. diphtheria oropharynx
C. peritonsillar abscess
D. Secondary syphilis
E. Chancre - amygdala .
108. Patient K., 34 years old, complains of an ulcer on the hard palate , enlarged lymph nodes,
weight loss . Objectively : for solid dark blue sky formation that rises above the level of mucosal
ulcer surface , slightly painful on palpation. Preceded the appearance of ulcers black , flat spots .
Your preliminary diagnosis ?
A. pemphigus vulgaris
B. gummy sifilid
C. lupus
D. Kaposi's sarcoma
E. fibroma
109. The patient K., 28 years old, on the lower lip ulcer size 0,7 x0, 8 cm in diameter on the
background of edema. Her appearance was preceded by hyperemia . Ulcer oval, painless, dense,
with raised edges . Submandibular lymph nodes densely - elastic consistency , mobile, painless , not
welded to one another. What is the most likely diagnosis ?
A. Necrotizing ulcerative stomatitis
B. cancerous ulcer
C. tuberculous ulcer
D. primary syphilis
E. tertiary

110. Patient N. , 55 years old , complains of paroxysmal pain in the right side of the face , rash in
the mouth and on the skin. Objective : to dramatically congested skin lip and chin on the right
multiple bubbles with transparent content . In the mouth, on the right cheek mucosa hyperemic
drain erosion covered with fibrinous coating. What is the preliminary diagnosis ?
A. allergic stomatitis
B. pemphigus vulgaris
C. Chronic recurrent herpes
D. shingles
E. erysipelas

111. A. The patient complains of dry lips , the presence of these crusts , which are exfoliated and
periodically reappear . OBJECTIVE: on the red border of the lower lip are massive korochnye
bedding , which is exposed after removing flushed surface covered with milky exudate . Place a
preliminary diagnosis.
 
A. atopic cheilitis
B. Exfoliative cheilitis, dry form
C. Weather cheilitis
D. Allergic contact cheilitis
E. Exfoliative cheilitis, exudative

112. Patient D. , 63 years old, complains of the appearance on the lateral surface of the tongue
ulcers. Ulcer appeared a month ago. The patient was treated with herbs. Has cardiovascular disease
over 15 years. When viewed from the side surface of the ulcer crater- left language forms with soft
edges , covered with a whitish bloom. Inflammation around the ulcer is absent. Diagnose .
A. trophic ulcer
B. Dekubitalnaya ulcer
C. tuberculous ulcer
D. cancerous ulcer
E. syphilitic ulcer
113. Patient K. , 52 years old, during the inspection of the mucous membrane of the cheeks ,
tongue, soft palate dentist revealed numerous ulcers with irregular podrytymi edges that tend to
spread. Bottom granular ulcers , covered with yellow- gray patina on the periphery found
microabscesses . Lymph nodes are enlarged , soldered into packets , elastic on palpation. Diagnose .
A. trophic ulcer
B. lupus
C. Milliarno ulcerative tuberculosis
D. Dekubitalnaya ulcer
E. tertiary

114. Patient N. , 28 years old . , An objective examination revealed a crack in the left corner of his
mouth . When opening the mouth crack looks oval erosion meat - red color with smooth edges ,
painless on palpation. Smooth bottom covered with a greasy coating. Left submandibular lymph
nodes are enlarged, chondroid consistency , painless. The mucous membrane of the oral mucosa
and the vermilion border - without pathological changes . Diagnose .
A. Streptococcal angular cheilitis
B. Ariboflavitaminoz
C. Mycotic angular cheilitis
D. primary syphilis
E. Chronic crack lips

115. Patient 45 years old, complained of education on the lower lip . On examination, the red border
of the lower lip is found grayish- red nodule with a hollow in the center , filled with horny masses ,
which can be easily removed . Nodule painless, movable . Place a preliminary diagnosis.
A. Narrow premalignant hyperkeratosis red border .
B. Papilloma .
C. Warty nodular precancer red border .
D. Basal cell carcinoma .
E. Keratoacanthoma .
116. 30 years old patient complains of severe pain during eating, bad breath , general weakness ,
fever . On examination : the patient pale, adynamic . Interdental papillae and marginal gingiva
hyperemic , edematous, with ulcers that are covered with necrotic plaque. In the analysis of blood :
er . - 4.8 × 1012 , HB - 130g / l , CPU - 0.9 leyk. -10 , 5x10 9 ( Yu -2 % ; ben 1% E- 5% p / Ya -8
% ; s/ya-47 % l -35 %); tromb. 250h109 - ESR -20mm / h What disease is a symptom of gum
disease in this patient ?
A. chronic leukemia
B. acute leukemia
C. agranulocytosis
D. Acute herpetic stomatitis
E. Necrotizing stomatitis
117. B. Patient complains of bleeding gums, nosebleeds , fatigue . Objectively: the lips, tongue,
bright red color . Oral mucosa dark cherry color. Cooperman positive symptom . Analysis of blood :
red blood cells - 6.8 ( 1012 / l, hemoglobin - 160 g / l, ESR - 2.1 mm / h ) . What are the probable
diagnosis :
A. Addison's disease - Biermer
B. disease Verlgofa
C. Vakeza disease ( erythremia )
D. acute leukemia
E. agranulocytosis
118. Male 56 years old complains of having " ulceration " in the mouth, which found a week ago.
OBJECTIVE: mucosal right retromolar area defined sharply demarcated lesion stagnant red velvet
surface size 2 * 1.5 cm Hearth sinks slightly on the surface defined areas of hyperkeratosis .
Histologically : poorly defined hyper-and parakeratosis , acanthosis in the thorny layer - cellular
and nuclear polymorphism, " horn pearls ." What is the most likely diagnosis ?
A. Lupus erythematosus, erosive form
B. Bowen's disease
C. Lichen planus, erosive form
D. Leukoplakia , erosive form
E. Keratinizing squamous cell cancer.
119. Female 52 years old complains of a burning sensation and tingling tongue, disappearing during
the meal , but growing by the end of the day , dry mouth , taste disturbance . First similar sensations
appeared two years ago after a trauma . Anatsidny suffering from gastritis . Objectively: general
condition is satisfactory , the patient is restless , tearful . GPRS - a pale pink color , dry , filiform
papillae on the dorsum of the tongue smoothed. Pharynxpharyngeoesophageal curtain - jerk sharply
reduced. What is the most likely diagnosis ?
A. Gangliitis sublingual ganglia
B. Glossitis Moeller - Günter
C. Glossodiniya
D. Chronic atrophic candidiasis, glossitis
E. desquamative glossitis
120. M. , 43 years old, complains of general weakness , fever , bleeding gums, bad breath .
Objectively: the oral mucosa and gums pale gums bleed when touched . Palatal the arc on the right -
the ulcer size 1x1 , 5 cm with jagged edges , covered with necrotic plaque , with no signs of
inflammation. In the blood test : Hb - 100 g / l , erythrocytes - 3,0 x10 12 / L, cs - 1,0 leukocytes -
250h10 9 / l nedif . blasts - 4%, promyelocytes - 3% myelocytes - 25% , young - 15%, p / - 8% , w /
I - 35 % , basophils - 2% , eosinophils - 3 %, lymphocytes - 5 % , platelets - 150h10 9l . What is the
most likely diagnosis ?
A. chronic lymphocytic leukemia
B. agranulocytosis
C. acute leukemia
D. thrombocytopenic purpura
E. chronic myelogenous leukemia
121. Man '31 ill 3 weeks ago, when there were general weakness , bone pain , intermittent fever,
bleeding gums when biting food and brushing teeth . OBJECTIVE: oral mucosa pale gums bleed
when touched with the slightest . In the blood test : Hb - 40 g / l , erythrocytes - 3.0 × 1012 / L, cs -
0.9 , leukocytes - 25h109 / l nedif . blasts - 60%, p / - 5 % , w / I - 15 %, lymphocytes - 20%, ESR -
60 mm / hour , platelets - 120h10 9 / l. What is the most likely diagnosis ?
A. agranulocytosis
B. acute leukemia
C. thrombocytopenic purpura
D. chronic myelogenous leukemia
E. chronic lymphocytic leukemia
122. Male 42 years old complains of bleeding , gingival overgrowth , bad breath , pain GPRS .
Objectively: the crowns of the teeth are covered by half loosened , bleeding gingival margin.
Submandibular, cervical lymph nodes are enlarged . In the analysis of blood leukocytes - 62.0 * 109
/ l , p / - 1% , w / I - 10% , eosinophils - 2% , monocytes - 5 %, lymphocytes - 82 % , ESR - 40
mm / hour. What is the most likely diagnosis ?
A. chronic lymphocytic leukemia
B. chronic myelogenous leukemia
C. acute leukemia
D. kissing disease
E. agranulocytosis
123. Patient D. , aged 35, complains for unusual formations in the mouth. OBJECTIVE: mucosal
tonsils, soft palate , on the line between the teeth - dense , painless , gray-white papules up to 4
mm , surrounded by a rim of hyperemia . When erosion occurs rubbing spatula meat red. On the
back of the tongue portions desquamation oval , shiny, below the level of the mucosa. Install the
diagnosis:
A. Secondary syphilis
B. leukoplakia
C. gummy sifilid
D. Lichen planus
E.Krasnaya erythematosus

124. The patient L., 28 years old, found on the lower lip erosion bright red , painless with chondroid
infiltrate at the base , 8 mm in diameter. Erosion appeared after the previous flushing. Regional
lymph nodes plotnoelasticheskoy consistency , painless, not soldered together . What is the most
likely diagnosis ?
A. primary syphilis
B. magnetoelectric effect
C. warty precancer
D. Necrotizing ulcerative stomatitis
E. tertiary

125. Candy factory worker , aged 50, complains of severe pain, burning and dryness of the oral
mucosa during the week . OBJECTIVE: GPRS flushed dry . Language bright red color , smooth
surface on the side is marked with the teeth in the folds , a small amount of white residue .
Vermilion border hyperemic, swollen, covered with thin scales , sometimes mucous macerated .
Preliminary diagnosis :
A. Acute atrophic candidiasis
B. allergic stomatitis
C. Chronic atrophic candidiasis
D. actinic cheilitis
E. Allergic contact cheilitis

126. Patient M. , 32 years old, complains of deterioration of general condition , increase in cervical
and submandibular lymph nodes , pain when swallowing . On examination : tongue coated gray-
white bloom , opening sharply bloodshot , hyperplastic tonsils . Marked catarrhal stomatitis . In
clinical analysis of blood - leukocytosis , the appearance of plasma cells and atypical mononuclear
cells . Diagnose :
A. kissing disease
B. Simanovskiy - Vincent Angina
C. AIDS
D. acute leukemia
E. agranulocytosis

127. By dentist patient appealed , 28 years old . After the clinical and additional tests was diagnosed
with necrotizing ulcerative stomatitis Vincent . What changes a blood test for this disease is
characterized by :
A. Neutrophilic shift to the left , increased erythrocyte sedimentation rate
B. Neutrophilic shift to the right , increased erythrocyte sedimentation rate
C. Eosinophilia, increased erythrocyte sedimentation rate
D. Monocytosis , increased erythrocyte sedimentation rate
E. Leukopenia, increased erythrocyte sedimentation rate
128. Patient P. , aged 17, complaining of constant dryness, burning , peeling lips , as well as cracks
in them, especially in the corners of the mouth . On examination revealed lihenifikatsiya lips and
eskoriatsii on the face and elbow creases . What is the most likely diagnosis :
A. eczematous cheilitis
B. exfoliative cheilitis
C. atopic cheilitis
D. actinic cheilitis
E. Weather cheilitis
129. Patient 19 years old, complained of a painless spots on the tongue . Objectively: the back of
the tongue on the gray- white coating , desquamation land surrounded by areas of hyperkeratosis .
Ill gastroenteritis. Diagnose :
A. glossalgia
B. Diamond language
C. pleated language
D. hairy tongue
E. desquamative glossitis
130. Patient K. , 55 years old , appealed to the dentist for a baseline medical examination . In the
history of 10 years suffers from gastric ulcer . Objectively: the oral mucosa is pale yellowish color ,
there are petechial hemorrhages . Back of the tongue is smooth, shiny , on the tip of the tongue
painful areas of desquamation of bright red color . Blood test: red blood cells 2 and 2 * 1012 / L
Hemoglobin -100g / L , the CPU -1 , 5, thrombocytes - 200 * 109 / L , WBC , 7.0 * 109 / l. What
are the most likely diagnosis :
A. agranulocytosis
B. iron deficiency anemia
C. disease Vakeza
D. Addison's anemia - Biermer
E. disease Verlgofa

131. An appointment with the dentist asked the patient M. , 19 years with complaints of increasing t
body , runny nose , headache, inability to eat a meal . Objective : to GPRS - punctulate enanthema
mucous membrane of the tonsils and soft palate is bright red . Tongue dry , shiny, bright red ,
threadlike papillae absent and mushroom - enlarged - " strawberry tongue ." On the skin -
melkotochechnye bright red rash.
A. Stomatitis in scarlet fever
B. allergic stomatitis
C. Stomatitis with flu
D. diphtheria stomatitis
E. thrombocytopenia

132. Patient 49 years in the treatment of lichen planus ( erosive and ulcerative form ) for 1 month
failed to reach epithelialization sections defeat. What is the best tactics of the doctor in this case?
A. Surgical treatment of an urgent histological examination
B. Purpose UHF , laser
C. Repeated treatment with the use of other means
D. Appointment imunostimuliruyuschey therapy : vitamins
E. Obkalyvanie lesion honsuridom
133. The patient was 32 years old, after a thorough examination was diagnosed as black " hairy "
tongue. What drugs do you think you want to assign for this disease ?
A. keratoplasticheskie
B. keratolytic
C. vitamins
D. antiseptic
E. antiviral

134. Patient 40 years old, contacted the clinic with complaints of pain when eating , putrid breath ,
fever up to 37.70 C. OBJECTIVE: on the gums of the lower jaw in retromolar ulcer area covered
with necrotic touch dirty-gray color , which is difficult to remove , exposing the painful bleeding
surface. Ulcers jagged edges , soft, surrounding tissues are swollen . Lymph nodes enlarged,
painful. Call microorganisms causative agent ?
A. Symbiosis actinomycetes and spirochetes .
B. Symbiosis spindle rods and spirochetes .
C. Symbiosis saprophytes and spirochetes .
D. Vincent pallidum .
E. Trichomonas .

135. Male 27 years old complains of severe pain in the mouth, fever up to 38.0 C, headache, joint
pain, general weakness. Sick three days ago, when after hypothermia , there are common
symptoms. Took sulfadimetoksin . Night ago appeared in the mouth and rashes on the skin.
Objectively: the skin of the hands and forearms - erythematous patches up to 1.5 cm in diameter and
with a bluish center sinks and bubbles with hard tire . On the red border of lips - bloody crusts ,
redness and swelling of the conjunctiva . On the oral mucosa - on the background of diffuse
hyperemia - large confluent erosion covered with a grayish bloom , sharply painful when touched .
Nikolsky negative symptom . What is the most likely diagnosis ?
A. Stevens-Johnson syndrome .
B. Medical stomatitis .
C. Exudative erythema multiforme .
D. Lyell's syndrome .
E. Acute herpetic stomatitis , severe .

136. Woman 60 years , complains of a burning sensation and slight pain in the soft palate.
Considers himself sick over a month. Self had no effect . Objectively: the mucous membrane of the
soft palate is determined erosion of irregular shape , measuring 2 x 3 cm , bright red color .
Surrounding mucosa pale . When rubbing a cotton swab appears detached mucosal epithelium at the
site unaltered . Patient enjoys dentures . What is the primary element lesions characteristic of this
disease.
A. subepithelial bubble
B. papule
C. intraepithelial bubble
D. bubble
E. blister
137. Patient K. , 52 years old, during the inspection of the mucous membrane of the cheeks ,
tongue, soft palate dentist revealed numerous ulcers with jagged edges podrytymi , which tend to
grow creeping along the periphery . And bottom edges of ulcers are granular in nature, covered with
yellow- gray patina , on the periphery of the grain found Trela . Lymph nodes are enlarged ,
soldered into packets , elastic on palpation. Diagnose .
A. leprosy
B. lupus
C. Milliarno ulcerative tuberculosis
D. Kollikvatsionny tuberculosis
E. tertiary

138. A patient , aged 49, glassblower , was diagnosed with precancerous warty red border of the
lower lip . What is the prognosis ?
A. self-healing
B. Malignancy within 6 months from the onset
C. Malignancy after 1-2 months of onset
D. Chronic course without malignancy
E. Recovery after a course of conservative therapy
139. The patient , 43 years old, complains of dryness in the mouth . On examination, her face pale ,
swollen upper eyelids sharply , lips and nose thickened. In oral language greatly increased in size ,
on a side surface of the teeth imprints . Teeth with caries amazed multiple localization in the
cervical area . What expert advice is needed to determine the treatment plan ?
A. endocrinologist
B. haematologist
C. gastroenterologist
D. neuropathy
E. allergist

140. B. The patient complains of spontaneous bleeding gums , nosebleeds . Objectively: the oral
mucosa is pale, swollen , thinned epithelium . Single identified erosion, as well as areas of
hemorrhage in the mucosa and positive symptom harness . Blood test: red blood cells - * 4.5 1012 /
L Hemoglobin - 120 g / l , the CPU - 1.0 leukocytes - 9 * 10 9 / L, 150 * 109 platelets / l. Your
diagnosis :
A. disease Velgofa
B. disease Vakeza
C. Bowen's disease
D. Biermer's disease
E. Henoch disease

141. Woman , 45 years old , a milkmaid , complains of weakness , chills, high fever . Notes muscle
pain , back pain, decreased appetite, dry and burning sensation in the mouth. On the oral mucosa are
cystic lesions with clear content , lips peel . Similar vesicles were also detected on the interdigital
spaces hands. Preliminary diagnosis :
A. murrain
B. Acute herpetic stomatitis
C. chickenpox
D. drug allergy
E. Exudative erythema multiforme
142. Patient N. , 60 years old , complains of weakness , increased salivation , metallic taste ,
sensation of heat , throbbing pain . Gums swollen, giperemirovanna , covered with a grayish- white
smelly raid, there are sores. Diagnose :
A. mercurial stomatitis
B. lead stomatitis
C. bismuth stomatitis
D. Necrotizing ulcerative stomatitis Vincent
E. ulocace
143. Work chemical plant for the production of paints complain of a metallic taste in the mouth ,
unusual breath. OBJECTIVE: mucous gum swelling, congestive hyperemia at the gingival edge
around the necks of front teeth defined bluish- black strip . In the analysis of blood leukocytosis ,
poikilocytosis , basophilic stippling of red blood cells . Which of the following substances was the
cause of the disease in this patient?
A. Lead.
B. Bismuth .
C. Mercury.
D. Cadmium.
E. Zinc .
144. Male 27 years old complains of severe pain in the mouth, fever up to 38.0 C, headache, joint
pain, general weakness. Sick three days ago, after hypothermia . Objectively: the skin of the hands
and forearms - erythematous patches up to 1.5 cm in diameter and with a bluish center sinks and
bubbles with hard tire . On the red border of lips - bloody crusts , redness and swelling of the
conjunctiva . On the oral mucosa - on the background of diffuse hyperemia - large confluent erosion
covered with a grayish bloom , sharply painful when touched . Nikolsky negative symptom . What
is the most likely diagnosis ?
        A. Stevens-Johnson syndrome .
        B. Medical stomatitis .
        C. Exudative erythema multiforme .
       D. Lyell's syndrome .
       E. Acute herpetic stomatitis , severe .

145. Female 64 years old complains of recurrent rashes on different parts of the oral mucosa
bubbles that are opened when eating , causing discomfort , slow to heal , leaving no scars .
OBJECTIVE: on the left cheek mucosa closer to the bottom crease in transition - bubble size 2,0 X1
, 5 cm thick cap and serous contents . When pressed with a spatula will not be opened . What is the
most likely diagnosis ?
       A. acantholytic pemphigus
       B. Benign Pemphigus neakantoliticheskaya only GPRS
       C. Bullous pemphigoid Lever .
       D. Bullous lichen planus .
       E. Vesico - vascular syndrome .
146. Female 47 years old complains of constant burning sensation in the tongue, dry mouth . Three
months ago, was raised bridge of gold with the support of 16 and 13. Examination revealed a
slightly swollen tongue , tip and lateral surface hyperemia , papillary apparatus unchanged. In 36
and 37 amalgam fillings . What is the most likely cause of this condition ?
       A. Galvanic currents .
       B. Allergy to gold.
       C. Fungal infection .
       D. Mechanical trauma prosthesis.
      E. Neurological disorders .

147. Male 45 years old complains of dryness and soreness of the lower lip . On examination : lower
lip swollen, dry, covered with small scales and cracks. In the area defined by the Klein extended
mouth salivary glands as red dots , which stands out from the transparent content . The mucous
membrane of the lower lip bumpy . What is the most likely diagnosis ?
     A. Exfoliative cheilitis.
     B. Actinic cheilitis.
     C. Weather cheilitis.
    D. Eczematous cheilitis.
    E. Glandular cheilitis.

148. Female 38 years old complained of pain in the lower lip for six months , difficult eating and
talking. Sought help from the community, the assigned treatment was ineffective . When viewed
from the middle of the lower lip is found deep crack , red border, and exciting part of the mucous
membrane of the lips , covered with brown crust , which occurs after the removal of the bleeding.
The crack divorced infiltrated . Select the optimum in this case medical tactics .
     A. Lubrication corticosteroid ointment.
     B. Applications keratoplasty funds.
     C. Irradiation of helium-neon laser .
     D. Excision of the crack.
     E. Lubrication with antibiotic ointment .

149. On examination of the working of the chemical enterprise D., 38 years old, suffering from
chronic enterocolitis , smoker, on back of the tongue in the posterior third of the midline detected
portion devoid of papillae , measuring 2 x 1 cm, slightly compacted by palpation . What is the most
likely diagnosis ?
A. desquamative glossitis
B. Chronic atrophic candidiasis
C. Secondary syphilis
D. rhomboid glossitis
E. Glossitis Günter -Miller
150 . A patient , aged 49, glassblower , was diagnosed with precancerous warty red border of the
lower lip . What is the prognosis ?
A. self-healing
B. Malignancy within 6 months from the onset
C. Malignancy after 1-2 months of onset
D. Chronic course without malignancy
E. Recovery after a course of conservative therapy

151. Patient G. , aged 47, complained of pain and a burning sensation in the mouth. OBJECTIVE:
on buccal mucosa on the line between the teeth and in the corners of the mouth multiple erosions to
1.0-1.5 cm in diameter, polygonal shape , bright red color , located on the hyperkeratotic plaques.
Cytologically : squamous cell epithelium. Diagnose :
A. Lichen planus, erosive form
B. Exudative erythema multiforme
C. Leukoplakia , erosive form
D. Secondary syphilis
E. lupus erythematosus
152. Patient V. , aged 57, complained of a painless ulcer on the tongue. Suffering cardiopulmonary
failure severity 2 , developed after the operated lung tuberculosis . OBJECTIVE: on the right side of
the tongue ulcer with dense splayed edges, the bottom of which is covered with growths
melkososochkovymi and necrotic plaque. The sample with 1% aqueous solution of toluidine blue
positive. Diagnose :

A. cancerous ulcer
B. Dekubitalnaya ulcer
C. tuberculous ulcer
D. syphilitic ulcer
E. trophic ulcer

153. Patient B. , 77 years old , located in the cardiology department for coronary heart disease ,
notes the appearance maloboleznennogo language education . Objectively: the back of the tongue to
the left with the transition to the lateral surface of a vast shallow ulcer , soft, covered with fibrinous
coating. The patient enjoys partial dentures , teeth 46,47 destroyed. Lymph nodes were not
enlarged. What treatment will promote healing of the ulcer ?
A. treatment 47.46
B. Application keratoplasty
C. Application of anti-inflammatory agents
D. Treatment of cardiovascular disease

154. Patient S. , 22 years old, complains of a painless ulcer on the back of the tongue .
OBJECTIVE: regional lymphadenitis , polyadenylation . On the back of the tongue -
blyudtsepodobnymi ulcer with edges measuring 8 mm in diameter with an elastic infiltration in the
ground, painless, the surrounding tissue intact. Place a preliminary diagnosis.
A. Cancer of the tongue
B. Dekubitalnaya ulcer
C. primary syphilis
D. abscess language
E. tuberculous ulcer
155. Patient D. , 39 years old , went to a doctor for treatment of tooth 36 . During the inspection
found papular rash on the soft palate , tonsils, mucosa of the cheek . Surrounding mucosa intact.
Painless papules at poskablivanii appears erosive surface at the base of which there is a dense
infiltrate . What additional research needs to be conducted?
A. serum samples
B. Complete blood count
C. immunological research
D. allergological sample
E. revmoproby
156. Patient M. , 52 years old, complains of pain during eating , hoarseness . Sick for a month.
When viewed on an unmodified gum mucosa , soft palate and shackle detected erosion of bright red
color . Mucosa and peels easily injured when rubbing with the appearance of erosions . Your
preliminary diagnosis ?
A. Dermatitis herpetiformis Duhring
B. Buleznaya form planus
C. Neakantoliticheskaya pemphigus
D. pemphigus vulgaris
E. Chronic recurrent herpes

157. Patient M. , aged 37, complains about the presence of painful sores on the mucous membrane
of the mouth , which makes it difficult meal. Objectively: the mucous membrane of the soft palate
has a painful ulcer with podrytymi soft edged up to 0.5 cm in diameter. Bottom is covered with
yellow tubercles. Cytology revealed Langhans cells . Your diagnosis ?
A. tuberculous ulcer
B. syphilitic ulcer
C. Dekubitalnaya ulcer
D. trophic ulcer
E.Rakovaya ulcer

158. 69 years old patient complains of the appearance for 2 months ulcers language next to the
destroyed tooth. Was diagnosed - dekubital ulcer . What is the nature of regional lymph nodes ?
A. not palpable
B. Increased soldered with fabrics , not painful
C. Increased soldered into packets , painful
D. Enlarged , painful , moving
E. Enlarged, painless, chondroid consistency

159. Patient N. , 50 years old , complains of severe pain in the tongue. Examination revealed in the
mucosa of the cheek and tongue determined painful erosions surrounded by horny papules that
coalesce into a lacy pattern. What is the most likely diagnosis ?
A. acute candidiasis
B. Leukoplakia , erosive form
C. Exudative erythema multiforme
D. Lupus erythematosus, erosive form .
E. Lichen planus, erosive form .

160. Patient 40 years old, contacted the clinic complaining of painful sores on the tongue.
Objectively: the back of the tongue on the shallow ulcers of irregular shape with soft edges
podrytymi . Ulcers bottom uneven, grainy , gray- yellow color. Around the ulcers are small
abscesses . What is the most likely diagnosis ?
A. Gummy ulcer .
B. Cancer .
C. Lupus .
D. Miliary tuberculosis ulcerative .
E. Lupus erythematosus .
161. Patient 40 years old, contacted the clinic with complaints of pain when eating , putrid breath ,
fever up to 37.70 C. OBJECTIVE: on the gums of the lower jaw in retromolar ulcer area covered
with necrotic touch dirty-gray color , which is difficult to remove , exposing the painful bleeding
surface. Ulcers jagged edges , soft, surrounding tissues are swollen. Lymph nodes enlarged, painful.
Call microorganisms causative agent ?
A. Symbiosis actinomycetes and spirochetes .
B. Symbiosis spindle rods and spirochetes .
C. Symbiosis saprophytes and spirochetes .
D. Vincent pallidum .
E. Trichomonas .

162. Patient 25 years old, complained of the presence of ulcers on the tongue. OBJECTIVE: on the
side surface of the tongue ulcer oval, 1 cm in diameter with raised edges and tight - elastic infiltrate
at the base , palpation painless . Lymph nodes on the affected side increased , dense , painless. What
is the most likely diagnosis ?

A. cancerous ulcer
B. Miliary tuberculosis ulcerative
C.Afta Setton
D. Shankriformnaya pyoderma .
E. primary syphilis

163. Patient '21 is explicit manifestation of necrotic stomatitis in the frontal area of the jaws. For 3
months noted weakness , fever to 37.50 C, drastic weight loss , diarrhea. On examination, her face
pale , lymph nodes are enlarged , painless on palpation. Place a preliminary diagnosis.
A. AIDS
B. Acute leukemia.
C. Vincent stomatitis .
D. Hypovitaminosis C.
E. Agranulocytosis .

164. Patient 45 years old accountant of private enterprise , complains of dry lips , sometimes
burning, the appearance of scales , which she skusyvaet . OBJECTIVE: dry lips on the red border
has large scales resembling mica platelets , their edges raised. After removing the scales of erosion
not only marked hyperemia . What is the most likely diagnosis ?

A. Weather cheilitis.
B. Exfoliative cheilitis.
C. Eczematous cheilitis.
D. Allergic contact cheilitis.
E. Actinic cheilitis.

165. Patient K., 65 years , complains of burning and dryness of mucous membranes in the last 2.5
months at lamellar prosthesis , which is used by 4 years. Objectively: the mucosa of the hard palate
in the prosthetic bed hyperemic, swollen, dry, with a minor amount of white bloom . On the back of
the tongue papillae atrophy . Blood test - no change . In what direction diagnosis requires further
examination ?
A. Allergic contact stomatitis
B. Glossodiniya
C. leukoplakia
D. Chronic atrophic candidiasis
E. Lichen planus

166. Patient 30 years old, complained of the presence of painful education in the mouth.
Objectively: the mucosal surface of the lower lip is defined epithelial defect with clear oval outlines
the size of 0.3 x 0.5 mm, surrounded by a rim of hyperemia and covered with fibrinous coating.
Which element of the lesions in this patient?
A. excoriation
B. erosion
C. aphtha
D. ulcer
E. crack

167. Patient , aged 40, to confirm the diagnosis chronic hyperplastic candidiasis is necessary to
direct microscopic study . When you need to take a scraping on mushrooms ?
A. Empty stomach or 3-4 hours after a meal.
B. Empty stomach or 5-6 hours after a meal.
C. After rinsing with a solution of baking soda.
D. After rinsing with a solution of manganese.
E. After rinsing solution furatsilina .

168. Patient Ts , 27 years old complains of the appearance of rash in the mouth, on the lips and
skin , general weakness , fever . Sick for 3 years , recurrent notes in spring and autumn . Objective :
to hyperemic and edematous mucosa of the mouth - bubbles painful erosions covered with gray -
yellow fibrinous coating. On the lips - hemorrhagic crusts . On the face, neck and dorsum of the
hand maculo - papular rash. Select the most likely diagnosis ?
A. herpetic stomatitis
B. pemphigus
C. Exudative erythema multiforme
D. Bulezny pemphigoid
E. Duhring disease

169. Patient D. , 20 years old , was diagnosed limfadematozny makroheylit . Which of the
following is used as a topical treatment ?
A. paraffin
B. Bath of chamomile decoction
C. corticosteroid ointment
D. keratolytics
E. keratoplasty

170. Patient S. , 24 years old, contacted the clinic for the treatment of chronic middle caries 15.
Within 2 months of notes recurrent muscle pain , malaise , fever, polyadenylation . Blood test for
HIV had a positive result. Indicate in which red blood cells is the development of
immunodeficiency virus ?
A. eosinophils
B. basophils
C. lymphocytes
 D. neutrophils
 E. monocytes

171. Patient T. , 30 years old, single , appealed to the clinic with complaints of an unusual kind of
language . Education appeared two months ago , weight loss in recent months - 7 kg . OBJECTIVE:
on the side surface of the tongue in the projection 47 has a section whitish hyperkeratosis
resembling algae accumulation . Palpation painless. Polyadenylation . How order MH of Ukraine
most fully defined tactics of the doctor with the patient?
A. Order 120 - Prevention of viral hepatitis and HIV infection , fighting nosocomial infections .
B. Order 300 - on the prevention of serum hepatitis
C. Order 408 - on measures to reduce the incidence of viral hepatitis
D. Order 700 - an improvement of medical care and strengthening the fight against nosocomial
infection instructions sterilization dressing

172. 44 year old man directed therapist. Is receiving treatment for acute exacerbations of chronic
gastroenteritis. Complaints: burning sensation and pain in the tongue, loss of appetite , nausea ,
fatigue , memory impairment . Objectively: the skin of the face and neck are defined erythematous
scaly patches . On GPRS - alternating areas of hyperemia and normal mucosa . Tongue swollen ,
bright red , shiny, filamentous papillae are atrophied . Manifestation of the failure of any vitamin is
given clinical situation ?
A. vitamin PP
B. vitamin A
C. vitamin B1
D. riboflavin
E. vitamin C

173. In the patient , the power line installer complains of a feeling of tightness, formation of scales
on the lips in the autumn-winter period . Examination revealed a red border of lips unevenly
hyperemic infiltrated somewhere , covered with small scales. Diagnose :
A. Weather cheilitis
B. Exfoliative cheilitis, dry form
C. Actinic cheilitis, exudative
D. Actinic cheilitis, dry form
E. Allergic contact cheilitis

174. The patient , aged 55, complains of burning in the mouth, pain while eating , a raid on the
cheeks and tongue. In the history of prolonged antibiotic treatment for pneumonia . When viewed
on a congested mucosa tongue and cheeks white plaque, which heavily removed . What additional
research method is needed to clarify the diagnosis :
A. trial Kavetsky
B. serological
C. Allergic
D. Stomatoskopiya
E. microbiological

175 . The patient , 60 years old , complains of pain during eating , the presence of erosion in the
oral mucosa . First ill more than a month ago. Objective : to unmodified gum shell , soft palate large
erosion of bright red color . Mucous and peels easily injured when rubbing . Soreness insignificant.
Smears - cells found Ttsanka . Determine diagnosis.
A. pemphigus vulgaris
B. pemphigus vegetans
C. Benign Pemphigus neakantoliticheskaya
D. Exudative erythema multiforme
E. Bullous lichen planus

176. The patient , 62 years old, complains of recurrent blisters in the mouth. When viewed from the
side surface of the left side first bubble size 8 x 8 mm, with hemorrhagic content . Palpation
painless. Nikolsky negative symptom . Blood pressure 200/120 mmHg . What is the most likely
diagnosis ?
A. pemphigus vulgaris
B. angioma language
C. Vesico - vascular syndrome
D. Exudative erythema multiforme
E. Dermatitis herpetiformis Duhring

177. Patient A. , aged 55, complains of itching, burning, rashes on the skin of the neck , ears , as
well as pain and ulceration in the mouth on the cheeks , palate . Ill two months . ago , there was a
strong burning, itching and small blisters on the skin , a week felt the pain and burning sensation in
the mouth. From history we found that 2 years ago had an allergic reaction to potassium iodide .
Remove the drug and the appointment of antihistamines - all phenomena are rapidly decreased.
OBJECTIVE: when viewed from the soft palate , cheeks on both sides - merged erosion bright red ,
slightly painful. SB around erosions swelling, hyperemic . Nikolsky negative symptom . On the skin
of the neck , ears , polymorphic elements [ erythema , vesicles , crusts ] . Diagnose .
A. Exudative erythema multiforme
B. pemphigus vulgaris
C. Dermatitis herpetiformis Duhring
D. allergic stomatitis
E. Lichen planus [ pemfigoidnaya form]

178. Patient 25 years old foci of necrosis along the gingival margin on both jaws . Facial skin
sallow hue. Looks older than his years . Submandibular, chin , neck and axillary lymph nodes are
enlarged , painless, mobile . Leads a hectic life, using drugs . In the last month notes weakness,
constant increase in body temperature to 37,3-37,50 C, diarrhea, weight loss of 15 kg. What is most
likely the preliminary diagnosis?
A. AIDS
B. acute leukemia
C. hypovitaminosis C
D. gingivitis Vincent
E. agranulocytosis
179. Patient 38 years old, complains of pain in the mouth when eating and from different stimuli ,
poor overall health. OBJECTIVE: painful erosions on the oral mucosa , which is on the edge of tire
scraps bubbles. Surrounding mucosa moderately inflamed. Notes on the back skin erosion, painful
to touch clothing. What additional testing is necessary for definitive diagnosis ?
A. cytological study
B. virological
C. immunological research
D. Complete blood count
E. bacterial study

180 . Patient 18 years , complains of an unusual kind of language . OBJECTIVE: on the back and
side surface of the tongue deep longitudinal folds, which are well expressed in filamentous
papillae . Language soft , moderately increased in size. Diagnose .
A. Glossodiniya
B. glossalgia
C. pleated language
D. desquamative glossitis
E. rhomboid glossitis

181. The patient was 24 years old, after a thorough medical examination was diagnosed as
rhomboid glossitis . Specify the etiology of this disease.
A. Local irritants , gastrointestinal disease
B. Secondary syphilis
C. Disturbance of embryogenesis
D. Miliary tuberculosis ulcerative
E. Thyroid disease
182. 49 years old patient complains of pain in the mouth , difficult meal. IBS suffers . An objective
examination of : shortness of breath, swelling of the extremities. Unchanged on the right buccal
mucosa ulcer with jagged edges , covered with a grayish- white necrotic plaque , bad breath . What
is the most likely diagnosis ?
A. trophic ulcer
B. traumatic ulcer
C. tuberculous ulcer
D. cancerous ulcer
E. Necrotizing ulcerative stomatitis

183. Woman 60 years , complains of a burning sensation and slight pain in the soft palate.
Considers himself sick over a month. Self had no effect . Objectively: the mucous membrane of the
soft palate is determined erosion of irregular shape , measuring 2 x 3 cm , bright red color .
Surrounding mucosa pale . When rubbing a cotton swab appears detached mucosal epithelium at the
site unaltered . Patient enjoys dentures . What is the primary element lesions characteristic of this
disease ?
A. subepithelial bubble
B. papule
C. intraepithelial bubble
D. bubble
E. blister

184. Duty dentist caused infectious disease clinic hospital to the patient 36 years old, admitted to
hospital the day before. 390S body temperature , severe headache , muscle pain , impaired function
of the gastrointestinal tract - nausea , dyspepsia . Currently complains of burning in the mouth,
pain , difficult meal. OBJECTIVE: oral mucosa hyperemic, many vesicles of different sizes, painful
erosions expressed hypersalivation . In history : 2 days ago eat dairy products bought at the bazaar .
Preliminary diagnosis :
A. murrain
B. Acute herpetic stomatitis
C. Behcet's syndrome
D. pemphigus
E. Dermatitis herpetiformis Duhring

185. Female 24 years old complains of a sharp pain when speaking and eating , the presence of
ulceration in the mouth. Sick for 3 years. Periodically appear in the mouth sores that heal very
slowly . Overall condition is not violated . In history: chronic enterocolitis . Objectively: the
mucosal surface of the tongue on the left side is defined ulcer rounded shape with a diameter of
about 1.5 cm , soft-edged , sharply painful when touched . The bottom of the ulcer is covered with a
grayish bloom . In the corners of the mouth are determined scar changes . There are sharp edges of
the teeth. What is the most likely diagnosis ?
A. aphtha Setton
B. Dekubitalnaya ulcer
C. Behcet's syndrome
D. stomatitis Vincent
E. tuberculous ulcer

186. The reception was at the dentist , the patient 20 years on the red border of the upper lip on the
right identified small grouped vesicles with serous content and painful erosions that appeared this
morning. Diagnosed with chronic recurrent herpes. What medicines must appoint a patient in a
given period ?
A. keratoplasty
B. painkillers
C. antiseptics
D. antiviral
E. antibiotics

187. Patient 53 years old, complains of severe pain buccal mucosa and lateral surface of the tongue
to the left. OBJECTIVE: on buccal mucosa and tongue left painful erosions , bleeding when
touched . Around erosions on erythematous mucosa gray-white papules coalescing into a lacy
pattern. Diagnosed with lichen planus . What form of the disease in the patient ?
A. Exudative - hyperemic
B. Buleznaya
C. giperkeratoticheskaya
D. Erosive - Ulcer
E. typical
188. Patient 42 years old, on the mucous membrane of the cheeks , lips, palate, tongue located on
hyperemic background bubbles merging with thin scraps of erosion along the periphery of the
epithelium . Vesicles contain many eosinophils ( 8-50 %). Jadassohn sample is positive, negative
Nikolsky sign . Put likely diagnosis :
A. pemphigus vulgaris
B. Chronic recurrent herpes
C. Cystic form planus
D. Mnogoformanaya exudative erythema
E. Dermatitis herpetiformis Duhring

189. Patient 55 years old, complains of pain during the meal , the presence of erosions on the oral
mucosa , hoarseness . First ill more than a month ago. When viewed on an unmodified gum mucosa
, soft palate and shackle revealed large erosion bright - red. Mucosa is easily injured and exfoliate
with little friction with the advent of erosion. Minor pain . What are the most likely diagnosis .
A. Exudative erythema multiforme
B. pemphigus vulgaris
C. Buleznaya form planus
D. Benign Pemphigus neakantoliticheskaya
E. Neakantoliticheskaya pemphigus

190. 40 years old patient complains of availability of education on the lower lip . Objectively: the
red border on the lower lip education hemispherical shape, dense , measuring 4 x 5 mm above the
level of lip 3-4 mm . Covered with thin , do not remove the scales. Regional lymph nodes were not
enlarged . What is the most likely diagnosis
A. Cutaneous horn
B. Papilloma
S. premalignant hyperkeratosis
D. warty precancer
E. Keratoacanthoma

191. Patient 32 years old , the body temperature of 38.9 0C , weakness , difficult conversation ,
inability meal. For 4 years the disease is repeated in the autumn - spring. On congested and
edematous mucosa of the lips , cheeks - papules , vesicles , erosions covered with fibrinous coating,
gray color. Nikolsky negative symptom . Which of the following is likely diagnoses ?
A. Neakantoliticheskaya pemphigus
Dermatitis herpetiformis Duhring V.
S. pemphigus vulgaris
D. Exudative erythema multiforme
E. Acute herpetic stomatitis

192. Patient B., 49 years old, the liquidators of the Chernobyl accident , there is frequent nosebleeds
, bleeding gums, positive symptom " rope" . Pre- diagnosed - Verlgofa disease . Which of the
indicators of a blood test will testify in favor of this diagnosis ?
A hemoglobin level
B. reticulocyte count
C. The number of platelets
D. leukocyte
E. ESR

193. Patient B. , 40 years old , there is discoloration of the skin and oral mucosa to dark cherry color
, bleeding gums , paresthesia. Were analyzed and diagnosed with blood - erythremia . Changes are
indicators of general blood analysis is characteristic of this disease?
A. Reduction in the number of erythrocytes and thrombocytes
B. Increasing the number of leukocytes and hemoglobin
C. Increase in color index and leukocytes
D. Reducing the number of erythrocytes and leukocytes
E. Increase the number of erythrocytes and hemoglobin

194. 22 years old patient diagnosed - necrotizing stomatitis Vincent . Surface treatment of ulcers
necrotic plaque is removed with difficulty. What drugs should be used ?
A. antiseptics
B. means affecting skin
C. proteolytic enzymes
D. ectylotic
E. antibiotics

195. Male 45 years turned to a consultation with the dentist . Complained presence on the red
border of the lower lip changed area whitish , who said two weeks ago. Pain is not noted .
OBJECTIVE: on the red border of the lower lip on the right near the center of a limited portion of
sharply defined polygonal shape measuring 0.5-1.0 cm surface of the lesion is grayish- white in
color and covered with small scales firmly seated . Hearth sinks slightly , the surrounding tissue is
not altered. Palpation of the changed area is painless, seal at the bottom is not defined . What is the
most likely diagnosis ?

A. Restrictions premalignant hyperkeratosis


B. lupus erythematosus
C. candidal cheilitis
D. Lichen planus
E. Leukoplakia , a planar shape

196. Male 20 years complains of severe pain in the mouth, fever up to 38,5 ° C headache and joint
pain, general weakness. Sick three days ago after hypothermia . OBJECTIVE: on the red border of
lips hemorrhagic crusts on the oral mucosa on the background diffuse hyperemia - large confluent
erosions and ulcers covered with a grayish- white coating . Conjunctivitis . On the skin of the hands
and forearms erythematous patches up to 1.5 cm in diameter with bubbles in the center. What is the
most likely diagnosis ?
A. medical stomatitis
B. Laila syndrome
C. Stevens-Johnson syndrome
D. Exudative erythema multiforme
E. Behcet's syndrome
197. The patient, 64 years went to the doctor with complaints of a slightly painful sores on the side
surface of the tongue , which appeared more than 2 months ago. OBJECTIVE: on the side surface
of the tongue ulcer with raised edges around it infiltrated tissue . Determined increased soldered to
each other and surrounding tissues submandibular lymph nodes. Preliminary diagnosis ?
A. tuberculous ulcer
B. Dekubitalnaya ulcer
C. Ulcerous- necrotic blood diseases
D. Cancer of the tongue
E. Syphilis (chancre )

198. The patient , 56 years old went to the doctor with complaints of a few painful ulcers on the
lower lip , which appeared a few months ago . He was treated , but to no avail . Objectively: the red
border on the lower lip podrytymi ulcer with jagged edges , at infiltrating the base. Its bottom is
covered with gray necrotic plaque. Determined little enlarged lymph nodes movable chin . What is
the most likely diagnosis ?
A. Chronic crack lips
B. Syphilis (chancre )
C. Cancer of the lower lip
D. tuberculous ulcer
E. Dekubitalnaya ulcer

199. Patient N. , 38 years old , complains of discomfort when eating , fever , malaise, swollen
lymph nodes . The patient smokes , uses alcohol moderately . OBJECTIVE: submandibular,
cervical , axillary lymph node palpation increased, mobile , painless . On the hard palate dark spot 3
cm in diameter , painless on palpation. Which of the following is necessary to clarify the
diagnosis ?
A. Complete blood count
B. Blood test for RW
C. Revmaproby
D. A blood test for HIV
E. Blood sugar

200. Patient I. , 25 years old, single , in good health . Complains of an unusual kind of language , a
sense of discomfort . Objectively: the back of the tongue on the periphery areas found desquamation
round shape resembling dvuhkopeechnuyu coin , painless on palpation. Submandibular lymph
nodes are enlarged , firm , painless , not welded to the surrounding tissues . For consultation to what
expert should send the patient?
A. gastroenterologist
B. hematologist
S. Venereology
D. allergist
E. dermatologist
201. Patient M. , 25years old , complains of pain on swallowing , the presence of plaque and
cavities. In history: chronic gastritis . OBJECTIVE: GPRS hyperemic, edematous , copious
greenish patina on the soft palate , the tonsils , the tongue , 36 , 37 cavities average depth .
Bacteriological examination of plaque Leffler found many diplococci . What tactics doctor - dentist
in relation to this patient?
A. GPRS treatment , antibiotics
B. consultation otolyaringologa
C. consultation gastroenterologist
D. Treatment of caries, oral hygiene
E. Consultation dermatology venereology

202. Patient B. , 45 years old , miner , complained of a burning sensation and pain in the mouth
with different stimuli . Objectively: the mucous membrane of the cheeks on the line between the
teeth, on the lateral surface of the tongue , land keratosis , towering above the mucosa , forming a
wart with lobed erosive surface. Which of the following is most likely diagnosis ?
A. Leukoplakia , towering form
B. Leukoplakia , warty form
C. Lichen planus, erosive form
D. Leukoplakia , erosive form
E. Lupus erosive ulcerous form

203. Patient A. , 62 years old, complains of having " sores " on the lip formed two weeks ago, does
not cause any symptoms. Twice previously noted the formation of similar elements , gradually heal
without treatment. OBJECTIVE: on the red border of lips between the midline and the angle of the
mouth, unchanged background , two erosion d ~ 0,5 and 0,3 cm , slightly painful , soft on
palpation , clean . What is the most likely diagnosis ?

A. Abrasive prekantserozny cheilitis Manganotti


B. Actinic cheilitis exudative
C. Weather cheilitis
D. Actinic cheilitis dry form
E. Exfoliative cheilitis dry form

204. Patient M. , 52 years old , suffering from diabetes, complains of burning , dryness in the mouth
, pain when eating , isolation viscous saliva. For 12 years, uses removable plate denture . Objective :
to hyperemic , edematous mucosa tongue, cheeks and palate lean removable hard plaque, which is
exposed after the removal of erosive surface. What is the most likely diagnosis ?
A. Leukoplakia , erosive form
B. Acute atrophic candidiasis
C. Chronic atrophic candidiasis
D. planus exudative hyperemic form
E. planus , erosive and ulcerative form
 

205. Patient B. , 40 years old , complains of burning , itching of the lips, swelling and redness ,
which emerged after the fit of the prosthesis of chromium-cobalt -nickel alloy . When viewed lips
hyperemic , edematous . On the skin, lips and red fringe corners of the mouth vesicles with
yellowish serous exudate , soak , brown. What is the most likely diagnosis ?
A. Exfoliative cheilitis exudative
V. Eczematous cheilitis
C. atopic cheilitis
D. Actinic cheilitis ekksudativnaya form
E. Allergic contact cheilitis

206. Patient B. , 35 years old , complains of lip augmentation , itching , pain , fractures , bleeding in
speaking and eating . Illness began four days ago after using a new toothpaste. Objectively: the lips
and surrounding skin swollen , giperemirovanny . On the red border of lips peel , crack , flake.
What additional studies be carried out to establish the diagnosis ?

A. Stomatoskopiyu
B. Skin allergy tests
C. Complete blood count
B. Bacteriological examination
E. vitropression

207. Female 52 years asked for advice . When handling complains availability sharply painful "
sores " in the mouth, which appeared about a month ago . Sought help from the community, the
recommended treatment had no effect . From history revealed that the patient suffered from
hypertension Article II . and diabetes for 15 years. An objective study on the mucosa of both cheeks
on the line between the teeth found two irregularly shaped erosion size 0.7-1.5 cm and 0.5-1.0 cm
right left , painful on palpation around which to hyperemic and edematous grounds located small
whitish papules coalescing into an openwork pattern. What is the most likely diagnosis ?
A. Melkersson -Rosenthal syndrome
B. Stevens-Johnson syndrome
C. Behcet's syndrome
D. Grynszpan's syndrome
E. Ramsay - Hunt's syndrome

208. Male 55 years old, disabled worker II group complains presence of whitish layers on the left
buccal mucosa , which said 3 months ago. Hypoacid suffering from gastritis . Composed by a
neurologist . Objectively: the mucous membrane of the left cheek on the line between the teeth
closer to the corner of the mouth portion is defined as plotnovata bumpy growths whitish size 2 x1,
5 cm with clear margins , rising above the level of the mucosa , painless on palpation. Surrounding
mucosa lesion is not changed. Bits 34 , 35 , 36 are destroyed, there are sharp edges of the teeth .
Choose the most efficient method of local treatment in this disease.
A. antifungals
B. Oil solution of vitamin A
C. surgical excision
D. ectylotic
E. Corticosteroids

209. Female 36 years old complains of dryness and peeling lips for a month. Application indifferent
ointment is not effective. Objectively: the red border of the lower lip rich red color , moderately
infiltrated , covered tightly seated whitish- gray scales when trying to remove that there is soreness
and bleeding. On the periphery of the hearth - a clouding of the epithelium in the form of strips of
white in the center - land depressions . What is likely
diagnosis?
A. leukoplakia
B. Lichen planus
C. lupus erythematosus
D. candida cheilitis
E. exfoliative cheilitis

210. When you call the clinic female 33 years complained of a burning sensation back of the tongue
, dry mouth . Amplified at a burning irritating foods . Sick for a week. Eve suffered severe angina,
took antibiotics . An objective study : oral mucosa pale, swollen tongue . On the background of
erythematous and edematous mucosa language in the folds of the friability is determined by gray-
white film , and desquamation on the tip of the lateral surface of the tongue . Saliva thick, stretches
threads per spatula. What research is most useful to confirm the diagnosis ?

A. Microscopy scraping language


B. cytological study
C. Overall full blood count
D. Blood sugar
E. Skin testing for antibiotics

211. Patient A. , 47y.o. , complains gum and nasal bleeding. Skin and mucosa hyperemic with dark
cherry shade . Cooperman positive symptom . What a picture of peripheral blood characteristic of
this disease :
A. E 3.5 × 10 12 degrees / l 140 g Hb / l, 5 ESR mm / hour
B. E 5.5 x10 12 degree / L, Hb 150 g / l, ESR 12 mm / h
C. E 5.8 x10 12 degree / L, Hb 140g / l, ESR 12 mm / h
D. E 6.1 x10 12 degree / L, Hb 180g / L, erythrocyte sedimentation rate of 1 mm / h
E. E 4.5 × 10 12 degrees / l, 120 g Hb / l, 6 ESR mm / hour

212. A. The patient , 46 years , complaints of burning , pain when eating hot and spicy .
Objectively: in retromolar cheeks symmetrically on a background of erythema - horny polygonal
papules forming a lacy pattern in the center of the hearth - painful erosions covered with fibrinous
coating. Your preliminary diagnosis :
A. allergic stomatitis
B. herpetic stomatitis
C. leukoplakia
D. Chronic recurrent aphthous stomatitis
E. Lichen planus

213. The disease develops without premonitory phenomena manifested rash aphthas 1-2 , size 8-10
mm , rounded, surrounded by hyperemia and whisk furry yellow - gray. Relapses are frequent ,
without precise patterns. What disease is characterized by ?
A. Chronic recurrent aphthous stomatitis
B. herpetic stomatitis
C. thrush
D. papular sifilid
E. systemic lupus erythematosus

214. Patient L. , 21 years old , who applied to dental treatment on the back of the tongue ulcer
revealed a rounded shape with a diameter of 1 cm , with clear raised edges and deep inflammatory
infiltrate in the ground. The bottom is flat , clean, red meat . Regional lymph nodes chondroid
consistency mobile, painless . Ulcer , which arose a week ago, the patient is not worried. What kind
of doctor should think ulcer ?
A. Tuberculosis
B. syphilitic
C. cancer
D. trophic
E. traumatic

215. Patient B., for 5 years every autumn , the disease begins with fever, pain in the joints. On the
skin of the upper and lower extremities cyanotic - pink spots with a diameter of 1-2 cm with a
bubble in the center. On the oral mucosa against the backdrop of extensive erythema and edema -
Bubbles , bubbles, sharply painful erosions , hemorrhagic crusts on the lips . Nikolsky negative
symptom . In blood test: A 12 • September 10 g / l, 4,7 • E December 10 g / L ESR 14 mm / hr
Select the most likely diagnosis :
A. necrotizing stomatitis Vincent
B. Chronic recurrent aphthous stomatitis
S. Neakantoliticheskaya pemphigus
D. exudative erythema multiforme
E. Chronic recurrent herpes

216. Candy factory worker , 38 years old complains of severe pain, burning and dryness of mucous
membranes of the mouth during the week . On examination, oral mucosa hyperemic sharply dry .
Language is bright red , smooth on the side surfaces of the teeth imprints . Vermilion border
hyperemic, swollen, covered with fine gray scales . Mostly mucous macerated . Which of these
ointments should be used for the topical treatment ?

A. prednizolonovuyu
B. interferon
C. sintomitsinovaja
D. Kanestenovuyu
E. eritromitsinovaya

217. G. 19 years old patient complains of sore lips, especially when their interdigitation , the
presence of crusts. On the red border of lips from the area until mid- Klein red border peel
yellowish - brown color, which appears after removing bright - red smooth surface without erosions
. Mucosa in the area of Klein slightly hyperemic and edematous . Diagnose :

A. acantholytic pemphigus
B. Actinic cheilitis, exudative
C. Exfoliative cheilitis, exudative
D. eczematous cheilitis
E. Weather cheilitis

218. 26 years old patient complains of availability ulcers on the tongue, which saw two weeks ago.
To the doctor did not address, was treated independently, but to no avail. The examination on the
back of the tongue on the background is unchanged ulcer diameter of 7 mm with splayed edges.
Smooth bottom , meat - red. Palpation painless, compacted base . Submandibular lymph nodes
dense , painless. Preliminary diagnosis :
A. cancerous ulcer
B. tuberculous ulcer
C. trophic ulcer
D. primary syphilis
E. Dekubitalnaya ulcer

219. G. 29 years old patient complains of a single slaboboleznennoe tumor-like formations on the
hard palate , which appeared about a month ago . OBJECTIVE: deep ulcer bluish - red color
surrounded by a massive dense infiltrate, covered with small granulations at the bottom reveals
bone sequestration . Diagnose :
A. Cancer of the hard palate
B. Abscesses of the hard palate
C. tertiary
D. Traumatic ulcer complicated by osteomyelitis
E. Miliary - ulcerative tuberculosis

220. 45 years old patient complains of foreign body sensation on the tongue, with the severity of the
conversation, dryness in the mouth . OBJECTIVE: on back of the tongue - up to 4 mm, dark ,
filamentary papillae . What is the most likely diagnosis ?
A. rhomboid glossitis
B. acute glossitis
C. desquamative glossitis
D. pleated language
E. Black " hairy tongue "
221. Patient 55 years old, complains of the appearance of small erosions on the oral mucosa .
OBJECTIVE: on the soft palate , retromolar area , cheeks, floor of the mouth , lips , gums , throat
on apparently intact mucosa revealed erosion edge burst bubbles. The examination revealed Ttsanka
cells . What pathological process causes their appearance :
A. parakeratosis
B. acanthosis
C. Papilomatoz
D. acantholysis
E. hyperkeratosis

222. Patient A. , aged 56, complains of burning neuralgic pain in the left half of the lower jaw ,
which appeared two days ago and intense rash blisters on reddened skin and mucosa of the lower lip
on the left. This is accompanied by malaise, headache , a temperature of 38  C, left-hand
lymphadenitis. Diagnose :
A. murrain
B. pemphigus
C. shingles
D. allergic stomatitis
E. Acute herpetic stomatitis

223. Patient 55 years old, complains of burning , unpleasant sensation in the tongue, for 5 years
suffering hypoacid gastritis. What changes are most likely to occur first in the patient ?
A. folding language
B. Atrophy and flattening of papillae
C. hypertrophy of the papillae
D. Erosion on the lateral surface of the tongue
E.Oblozhennost language

224. Patient 26 years old, complains of an increase in temperature up to 380 C. , painful swallowing
, bad breath , bleeding gums . Ill with rheumatism , take phenylbutazone , aminopyrine .
OBJECTIVE: oral mucosa pale, hemorrhages , areas of necrosis on the tonsils and the gingival
margin. What changes in the blood , the patient most likely to occur ?
A. myeloblastosis
B. agranulocytosis
C. thrombocytopenia
D. neutropenia
E. Increase in color index .

225. Female 35 years old complains of periodic appearance of sores in the mouth for three years .
Relapse is repeated 4 to 5 times a year. Ulcers heal within 7-8 days . OBJECTIVE: on the side of a
language is defined elements defeat rounded shape of 0.5 mm in diameter with sharp edges are
covered with a grayish bloom , surrounded by a crown of hyperemia , painful when touched .
Diagnosis: chronic recurrent aphthous stomatitis . What is the most likely pathogenetic mechanism
of the disease ?
A. mechanical injury
B. Allergic reaction
S. Staphylococcal infection
D. autoimmune mechanism
E. Viral infection

226. Patient B., 30 years old, diagnosed with acute ulcer - necrotizing gingivitis, moderate severity.
Drug which group should be used in the final stage of treatment?
A. antibiotics
B. antioxidants
C. keratolytics
D. keratoplasty
E. sulfonamides

227. A young man of 18 years when applying to the clinic complaining of sore gums during
mealtimes . Acutely ill three days ago, when after hypothermia appeared low-grade fever and pain
in the gums . OBJECTIVE: sanitized mouth , gums on the upper and lower jaws hyperemic,
edematous , bleeds easily when touched tool. Teeth and gums are covered with copious amounts of
plaque. In the 21 , 22, 23 , 24 and 25 at the interdental papilla determined dirty gray patina , which
is formed after the removal of a bleeding ulcerated surface. Plaque removal sharply painful. What
additional research is needed to carry out this sick in the first place?

A.Osvidetelstvovanie HIV - infection


B.Mikroskopiyu plaque from the papilla
C.Obschy full blood count
D.Rentgenografiyu jaws
E.Analiz blood sugar

228. A patient 20 years complains of bleeding from the gums , cosmetic defect , general weakness ,
lethargy . OBJECTIVE: pale skin , submandibular lymph nodes are enlarged , soft , painless. Oral
mucosa anemic , tooth- papilla enlarged, hyperemic , friable , swollen , bleeding. In the anterior
surface of the teeth in okolonebnoy - papilla completely cover the crowns of the teeth. Blood test:
HB - 100 g / l of erythrocytes - 2,7 • October 12 l leukocytes - 22 • September 10 g / l, cs - 1.0 -0
myelocytes , band , c / i - 12 % lymphoblasts - 6 % lymphocytes - 18 % monocytes - 3% , platelets -
160 • September 10 g / L ESR - 50 mm / hr What is the primary tactic dentist?

A.Konsultatsiya endocrinologist
B.Lechenie periodontal pathology
C.Konsultatsiya hematologist
D.Konsultatsiya immunologist
E.Konsultatsiya therapist.

229. Female 52 years old complains of a burning sensation and tingling tongue , vanishing in
eating, but increasing towards the end of the day , dry mouth , impaired
mouthfeel . First similar sensations appeared two years ago after
trauma . Anatsidny suffering from gastritis . Objectively: general condition
satisfactory , the patient is restless , tearful . GPRS - pale pink
color , dry , filiform papillae on the dorsum of the tongue smoothed. Pharynx - curtain
reflex dramatically reduced. There is a violation of sensitivity to skin
segmental type . What is the most likely diagnosis ?
A Glossodiniya
B Glossitis Moeller - Günter
C sublingual ganglia ganglion
D Chronic atrophic candidiasis, glossitis
E deskvamativny glossitis
230. Male 29 years old complains of itching, burning, soreness of the lips, appearing and
disappearing in the summer in the rest of the year. Sick for 5 years. Objectively: the red border of
the lower lip hyperemic, edematous , against this background, defined small bubbles erosion
diameter and 3 mm , crusted , painful when touched , and a single crack . What is the most likely
diagnosis ?
A Actinic cheilitis, exudative
B Meteorological cheilitis
C Acute eczematous cheilitis
D Exfoliative cheilitis, exudative
E erosive lupus vermilion border
231. Female 55 years old complains of burning tongue , pain during eating,
taste disturbance , general weakness, headache. objectively:
pale skin on the back of the tip of the tongue and bright red spots,
painful on palpation. In the analysis of blood : er . - 2.2 * 1012 / l , Hb - 70 g / l, cs -
1.4 anisotropy Poikilocytosis l. - 4.2 * 109 / L, ESR - 10 mm / hour. With administration
Which drugs should be carried out the overall impact of the local
symptomatic treatment of this patient?
A Vit. B12 and folic acid
B of iron and vit. With
C aminocaproic acid and calcium chloride
D corticosteroid hormones
E Nicotinic acid and thiamine

232 . Female 44 years complains of pain in tongue when receiving spicy food , perversion
taste , smell , dryness in the oral cavity , the presence of cracks in the corners of painful mouth.
OBJECTIVE: oral mucosa pale, edematous , and on the back of the tip
language - atrophy of papillae in the corners of the mouth - painful bleeding cracks . In
analysis of blood : er . - 2.9 × 1012 / L , Hb - 80 g / l, cs - 0.7 , ESR - 10 mm / hour. On the
background of the overall impact of any preparations necessary to carry out the local symptomatic
treatment of this patient?
A supplementation with iron and vit. With
B Vit. B12 and folic acid
C Nicotinic acid and thiamine
Antifungal D and iodine preparations
E calcium gluconate and sodium fluoride
233. The patient appealed to the dentist complaining of frequent bleeding from the mucous
membranes of the mouth and nose. From history we found that the same problems were her father .
OBJECTIVE: There are multiple telangiectasias and angiomatoznye education on facial skin and
mucous membranes of the nose, cheeks, lips . Analysis of blood within normal limits. Diagnose .
A Addison's disease - Biermer . .
B Verlgofa disease .
C disease Vakeza
D Cushing's disease .
E Rendu Osler disease .
234. Patient 53 years diagnosed with leukoplakia smokers Tappeiner . what
pathologic process prevails in this histology
disease?
A Hyperkeratosis
B Acanthosis
C Dyskeratosis
D parakeratosis
E papillomatosis

235 Patient 16 years complained of pain in the mouth, sores ,


38 fever , headache . An objective examination of mucous
oral hyperemic, edematous . Mucosal erosions set merged with
polycyclic outlines covered grayish- white film ,
located on the hard palate , gums , lips . What is the most probable
preliminary diagnosis ?
A pemphigus vulgaris
B erythema multiforme exudative
C Acute herpetic stomatitis
D FMD
E Allergic stomatitis
236. Female 59 years old complains of burning and light pain in the soft
sky during the month. Enjoys dentures . Objectively: the soft
sky irregularly shaped erosion size 2x3 cm, bright red color .
Surrounding mucosa pale . When rubbing a cotton swab notes
apparently unaltered epithelium detachment around erosion. What is the most
probable preliminary diagnosis ?
A pemphigus vulgaris
B allergic reaction to plastic
C Bullous pemphigoid
Dermatitis herpetiformis Duhring D
E erosive stomatitis
237. Male 26 years complained of the presence of rash in the mouth.
OBJECTIVE: mucosal and soft palate arches defined papules size
about 1 cm in diameter , slightly protruding above the surface of the mucous coated
grayish bloom and surrounded by narrow hyperemic rim. at
poskablivanii plaque is removed with a spatula , then there is the red meat
color erosion . Submandibular nodes are enlarged , painless on palpation. what
additional methods of research necessary to this patient.
A microscopy and serological tests
B Fluorescent and biopsy
C microscopy and culture on nutrient medium
D Stomatoskopiya and cytology
E Cytology and luminescent
238. Patient , 23y.o., a convert to the first day of the disease , the diagnosis :
acute herpetic stomatitis moderate severity. What ointment
processing elements defeat you assign in the first place?
A Antiviral .
B Hormone .
C sulfa .
D Antibiotikosoderzhaschuyu .
E Fermentosoderzhaschuyu .
239. Female 52 years complains constant burning sensation on the tip and side
surface of the tongue . Burning occurs in the morning and the evening is enhanced ,
calms down when eating . Considers herself to be sick for about a year . availability
staging of the disease is associated with dentures . In history : anatsidny
gastritis, surgery for uterine fibroids , hypertension
IIstepeni , poor sleep , anxiety . OBJECTIVE: swollen tongue slightly , on the back
language - lean white coating , atrophy of the filiform papillae . The rest of the mucous
without obvious pathological changes. Pharyngeal reflex - curtain lowered.
What is the most likely diagnosis ?
A Glossodiniya
B allergic reaction to plastic
Glossitis Günter C -Miller
D Candida glossitis
E lingual nerve neuritis

240. Female 33 years complains of an increase of the upper lip . In the history of right-sided facial
nerve paresis . First ill a year ago, when there was a swelling of the upper lips , which quickly
disappeared , but remained somewhat larger lip . three days ago after hypothermia lip again sharply
increased . Objectively : the top
lip dramatically increased in size , more on the right , palpation is soft, elastic
painless, impressions from fingers remains. Swollen tongue , on back of the tongue -
folds, bumpy surface of the tongue . What is the most likely diagnosis ?
A Meiji Trofedema
B Angioedema
C Usher Syndrome
D Melkersson -Rosenthal syndrome
E elephantiasis ( elephantiasis ) lips

241. Female 36 years old complains of pain in the mouth, increasing weakness . on the eve suffered
a sore throat. Within 10 days of taking sulfadimetoksin . objectively:
oral mucosa is dry, slightly hyperemic, multiple small necrotic ulceration character on the buccal
mucosa , lips, tongue , along gingival margin . Gums swollen, bleed easily . Blood test: er . - 4.0 *
10 ** 12 / L, Hb - 140 g / l, cs - 1.0 , thrombus. - 200 * 10 ** 9 / L Lake . - 1.5 * 10 ** 9 / l , p / i -
0, c / I - 0.03 * 10 ** 9 / L (2 %) , e . - 0 , b . - 0 , lymph . - 1.455 * 10 ** 9 / L (98%) , Mon . -
0,015 (1% ) ESR - 50 mm / hour
Any symptom of the disease is necrotizing syndrome in this patient?
A Agranulocytosis
B acute leukemia
C Vincent stomatitis
D Chronic lymphocytic leukemia
E Chronic myeloid leukemia

242. Male 28 years old complains of a sharp pain when eating , the presence of
ulcerations in the mouth. Considers herself sick 5 years. Periodically cavity there are painful mouth
ulcers heal within 3-4 weeks . Objective : to mucosa of the lower lip on the left there is an ulcer
about 1 cm in diameter with soft edges , sharply painful on palpation. At the bottom of the ulcer
grayish- yellow coating . on the side
surface of the tongue are cicatrices . Vestibular position 23. Overall condition is not violated. What
is the most likely diagnosis ?
A Dekubitalnaya ulcer
B AFTA Setton
C Behcet's syndrome
D syphilitic ulcer
E Stomatitis Vincent
243. Patient S. , 40 years old , asked to reorganize . When viewed from the language found
numerous " furrows " extending from the deepest central " Furrow " . This state of the patient noted
since childhood. symptom of what syndrome may be present condition ?
A Melkersson -Rosenthal Syndrome .
B Syndrome Grynszpan .
C Sjogren's syndrome .
D Behcet's syndrome .
E Stevens-Johnson syndrome .

244. Patient I. , 25 years old, single , in good health . Complains of an unusual kind of language
discomfort . Objectively: the back of the tongue on the periphery found
land desquamation round shape resembling a coin dvuhkopeechnuyu painless on palpation.
Submandibular lymph nodes enlarged dense, painless, not spayanye to surrounding tissues. For
consultation to any specialist must send the patient?
A allergist.
B hematology.
C Gastroenterology .
D dermatology.
E Venereology .

245. Patient 42 years old, complains of burning , pain in the tip of tongue , impaired
taste sensitivity , dry mouth . In the history anatsidny gastritis . Objectively: numerous cracks at the
corners of the mouth, the side surfaces of the first prints teeth, back of the tongue red color on a
background of pale oral mucosa atrophy papillae on the tongue . In the history of blood : Hb 80 g / l
, Riyadh . 3.5 x 1012 / l ts.P.0 7 .
What treatment is likely to be appointed the patient ?
A supplementation with iron and B vitamins parenterally.
B Iron preparations and B vitamins orally.
C Blood transfusions , vitamin C orally.
D Vitamin B12 with folic acid orally.
E Vitamin B12 with folic acid parenterally.

246. Patient G. , 56 years old , complains of an increase of the lower lip and the pain of hot , sour,
salty, bitter , sticky lips in the morning. Lifelong periodically lip thickened , become lumpy . When
viewed from the lower lip increased. In
middle third zone Klein few small red dots with holes of
which show through the transparent drops . Red border dry , cracked and
erosions , shelled. What is the most likely diagnosis ?
A glandular cheilitis
B Ekfoliativny cheilitis
Actinic cheilitis C
D allergic contact cheilitis
E Atopic cheilitis.

247. Patient K., 52 years old , the driver complained of pain , shortness of
mouth opening . Was diagnosed with leukoplakia ulcerative form . After 10 -
day course of therapy appeared sluggish granulation. On the periphery of ulcers observed
strengthening processes keratinization. Which of the following must hold in
first?
A study Bacterioscopic
B Histological examination .
C Stomatoskopiyu .
D Bacteriological examination .
E cytology.

248. Patient N. , 34 years old, underwent a course of treatment of infectious - allergic form
exudative erythema multiforme . Comorbidities tonsillitis, periodontitis . Which of the following
will prevent a relapse ?
A Rehabilitation of foci of infection .
B hyposensitization therapy .
C Ultraviolet radiation .
D Corticosteroid therapy .
E Autohemotherapy .

249. The working aluminum plant , 55 years old , suffering from chronic cholecystitis
on the buccal mucosa close to the corner of the mouth formation lumpy white
Towering above the surrounding mucosa , with painless seal in basis . The oral cavity is not
sanitized . Which of the following is priority in the tactics of the dentist?
A referral to the Oncology Center .
B Purpose keratolytic agents.
C Conducting cryodestruction hearth.
D referral to a gastroenterologist.
E oral hygiene .

250 . Student 18 years complained of roughness , dryness of mucous


membranes of the mouth . Constantly skusyvaet not controlling , through the mucous
interdigitation . Suffers from chronic cholecystitis , smokes. GI - 2.3 . objectively:
mucosa on the line between the teeth loose, edematous , has whitish color.
Which of the following is the leading part in the emergence of
disease?
A smoking .
B Inadequate oral hygiene.
C Somatic pathology .
D xerostomia .
E Habitual biting GPRS .

ANSWERS TO TEST JOB IN SECTION "DISEASE GPRS"


1 С 51 Е 101 С 151 С 201 Е
2 Д 52 А 102 С 152 А 202 Д
3 А 53 А 103 В 153 Д 203 А
4 Е 54 С 104 С 154 С 204 С
5 В 55 С 105 А 155 А 205 В
6 А 56 А 106 С 156 Д 206 С
7 Д 57 А 107 Е 157 А 207 Д
8 А 58 А 108 Д 158 Д 208 С
9 Д 59 А 109 Д 159 Е 209 С
10 А 60 В 110 Д 160 Д 210 А
11 С 61 А 111 Е 161 В 211 Д
12 С 62 Д 112 А 162 Е 122 Е
13 Д 63 В 113 С 163 А 213 А
14 С 64 А 114 Д 164 В 214 В
15 С 65 А 115 Е 165 Д 215 Д
16 А 66 А 116 Е 166 С 216 Д
17 А 67 Е 117 С 167 А 217 С
18 Д 68 В 118 В 168 С 218 Д
19 Е 69 В 119 С 169 А 219 С
20 С 70 Д 120 Е 170 С 220 Е
21 А 71 С 121 В 171 А 221 Д
22 А 72 Д 122 А 172 А 222 С
23 Д 73 Д 123 А 173 А 223 В
24 В 74 Е 124 А 174 Е 224 В
25 Е 75 А 125 А 175 А 225 Д
26 В 76 В 126 А 176 С 226 Д
27 А 77 А 127 А 177 С 227 С
28 С 78 Д 128 В 178 А 228 С
29 С 79 Е 129 Е 179 А 229 А
30 С 80 В 130 Д 180 С 230 А
31 Д 81 Е 131 А 181 С 231 А
32 С 82 Д 132 А 182 А 232 А
33 А 83 Д 133 В 183 С 233 Е
34 А 84 Д 134 В 184 А 234 А
35 А 85 Е 135 А 185 А 235 С
36 С 86 С 136 С 186 Д 236 А
37 В 87 В 137 С 187 Д 237 А
38 А 88 С 138 С 188 Е 238 А
39 Е 89 А 139 А 189 В 239 А
40 С 90 Е 140 А 190 Д 240 Д
41 В 91 С 141 А 191 Д 241 А
42 С 92 Д 142 А 192 С 242 В
43 А 93 Е 143 А 193 Е 243 А
44 С 94 А 144 А 194 С 244 Е
45 Е 95 А 145 В 195 А 245 А
46 А 96 А 146 А 196 С 246 А
47 А 97 С 147 Е 197 Д 247 В
48 А 98 Д 148 Д 198 С 248 А
49 А 99 Д 149 Д 199 Д 249 А
50 С 100 С 150 С 200 С 250 Е

You might also like